You are on page 1of 58

CELL BIOLOGY

Posted: Wed Jun 08, 2005 12:05 pm Post subject: USMLE 1 MCQ5

1> A normal, healthy, 25-year-old man lives at the beach. His twin brother has
been living in a mountain cabin for the past 2 years. Which of the
following indices would be expected to be higher in the man living at
sea level?
A. Diameter of pulmonary vessels
B. Erythropoietin production
C. Mitochondrial density in a muscle biopsy
D. Renal bicarbonate (HCO3-) excretion
E. Respiratory rate
Answer
--------------------------------------------------------------------------------
2>A woman presents to a dermatologist because she has lost almost all the
hair on her body, including scalp hair, eye brows, eye lashes, arm pit and
groin hair, and the fine hairs on her body and extremities. She most likely
has a variant of which of the following?
A. Alopecia areata
B. Androgenic alopecia
C. Chronic cutaneous lupus erythematosus
D. Lichen planopilaris
E. Trichotillomania
Answer
--------------------------------------------------------------------------------

3>The greater omentum is derived from which of the following embryonic


structures?
A. Dorsal mesoduodenum
B. Dorsal mesogastrium
C. Pericardioperitoneal canal
D. Pleuropericardial membranes
E. Ventral mesentery
Answer
--------------------------------------------------------------------------------

4>A 34-year-old woman is prescribed an antidepressant, which she has taken


for the past 3 months. She is on no other medications and is in generally
good health. After attending a party, at which she consumed wine and
cheese, she is rushed to the emergency room with tachycardia, headache,
and a blood pressure of 200/100. Which antidepressant is she most likely
taking?
A. Amitriptyline
B. Fluoxetine
C. Phenelzine
D. Sertraline
E. Trazodone
Answer
--------------------------------------------------------------------------------

5>Which of the following hormones is secreted by anterior pituitary cells


that stain with acidic dyes?
A. ACTH
B. FSH
C. LH
D. Prolactin
E. TSH

6>A 23-year-old male graduate student gets into a serious argument with
one of his college professors, making a physical threat to the professor,
CELL BIOLOGY

and necessitating a call to campus security. The argument was precipitated


by an incident between the professor and the student's girlfriend; when
the professor corrected the student's girlfriend in class, the student
felt the professor was verbally abusive. Which of the following is
the most likely diagnosis ?
A. Dependent personality disorder
B. Histrionic personality disorder
C. Narcissistic personality disorder
D. Paranoid personality disorder
E. Passive aggressive personality disorder
Answer
--------------------------------------------------------------------------------

7>Which of the following would shift the oxygen-hemoglobin dissociation curve


to the right?
A. Carbon monoxide poisoning
B. Decreased PCO2
C. Decreased pH
D. Decreased temperature
E. Decreased 2,3-DPG
Answer
--------------------------------------------------------------------------------

8>The wife of a 48-year-old male patient brings him to the emergency room and
says that his memory has progressively gotten worse over the last several
years. She also says his personality has been changing. The physician notes
abnormal writhing movements of the man's limbs and hyperreactive reflexes
. MRI reveals a loss of volume in the neostriatum and cortex. This disease
is inherited via an
A. autosomal dominant trait
B. autosomal recessive trait
C. x-linked dominant trait
D. x-linked recessive trait
Answer
--------------------------------------------------------------------------------

9>A man presents to a dermatologist because of a severe mucocutaneous rash


that involves most of his body, including his palms and soles. Questioning
reveals that he is a merchant marine who several months previously had an
encounter with a prostitute in Southeast Asia. Which of the following is
the most likely causative agent of this rash?
A. Herpes simplex I
B. Herpes simplex II
C. HIV
D. Neisseria gonorrhoeae
E. Treponema pallidum
Answer
--------------------------------------------------------------------------------

10>A patient with familial hypercholesterolemia undergoes a detailed serum


lipid and lipoprotein analysis. Studies demonstrate elevated cholesterol
in the form of increased LDL without elevation of other lipids.
This patient's hyperlipidemia is best classified as which of the
following types?
A. Type 1
B. Type 2a
C. Type 2b
D. Type 3
E. Type 5
CELL BIOLOGY

Answer
--------------------------------------------------------------------------------

11>A mutation affecting the development of the diencephalon could interfere


with the secretion of which of the following hormones?
A. Adrenocorticotrophic hormone (ACTH)
B. Epinephrine
C. Oxytocin
D. Prolactin
E. Thyroid stimulating hormone (TSH)
Answer
--------------------------------------------------------------------------------

12>Which of the following organisms is the most common cause of


community-acquired pneumonia?
A. Chlamydia pneumoniae
B. Haemophilus influenzae
C. Mycoplasma pneumoniae
D. Staphylococcus aureus
E. Streptococcus pneumoniae
Answer
--------------------------------------------------------------------------------

13>A 54-year-old male with acute lymphocytic leukemia develops a blast crisis.
He is treated with intensive systemic chemotherapy. Following treatment,
the patient will be at increased risk for the development of
A. bile pigment gallstones
B. cholesterol gallstones
C. cystine kidney stones
D. struvite kidney stones
E. uric acid kidney stones
Answer
--------------------------------------------------------------------------------

14>A 65-year-old man with mild heart failure is treated with a loop diuretic
. A few days later the man complains of muscle weakness. Laboratory results
are shown below. Arterial PCO2: 48 mm Hg Arterial pH: 7.49
Plasma HCO3-: 35 mEq/L Which of the following is most likely
decreased in this man?
A. Plasma aldosterone
B. Plasma potassium
C. Potassium excretion
D. Renin secretion
E. Sodium excretion
Answer
--------------------------------------------------------------------------------

15>A 27-year-old white male presents with a 3-week history of several swollen
and painful toes and knees. He has a past history of conjunctivitis. He
also describes some low back stiffness that is more severe in the morning.
Which of the following is the most likely diagnosis?
A. Gout
B. Lyme disease
C. Reiter's syndrome
D. Rheumatoid arthritis
E. Septic arthritis
Answer
--------------------------------------------------------------------------------
CELL BIOLOGY

16>A 37-year-old woman who was diagnosed with AIDS 3 years earlier is unable
to work, is physically debilitated, and requests her physician to provide
her with medications with which to take her own life. The most common
emotional disorder that results in such requests by patients is
A. bipolar I disorder, manic type
B. borderline personality disorder
C. factitious disorder
D. major depressive disorder
E. schizophrenic disorder
Answer
--------------------------------------------------------------------------------
17>
A 25-year-old man presents with a chief complaint of persistent,
high-pitched ringing noises in his ears. Questioning reveals that
he has also been losing his balance lately. CT of the head demonstrates
bilateral tumors involving the vestibulocochlear nerve. Which of the
following chromosomes contains the tumor suppressor gene most likely to
be involved in this case?
A. 5q
B. 13q
C. 17q
D. 18q
E. 22q
Answer
--------------------------------------------------------------------------------

18> An autopsy is performed on a man who suddenly began vomiting voluminous


quantities of blood and exsanguinated. Which of the following organisms is most
likely implicated in the pathogenesis of this disease?
A. Cryptosporidium parvum
B. Entamoeba histolytica
C. Escherichia coli
D. Helicobacter pylori
E. Mycobacterium tuberculosis
Answer
--------------------------------------------------------------------------------

19>During embryological development, hematopoiesis occurs in different organs


at different times. Which of the following are the correct organs, in the
correct sequence, at which hematopoiesis occurs embryologically?
A. Amnion, yolk sac, placenta, bone marrow
B. Placenta, liver and spleen, yolk sac, bone marrow
C. Placenta, spleen and lymphatic organs, bone marrow
D. Yolk sac, bone marrow, liver and spleen
E. Yolk sac, liver, spleen and lymphatic organs, bone marrow
Answer
--------------------------------------------------------------------------------

20>A 27-year-old male is brought into the emergency room by the police,
who found him walking aimlessly, shouting the names of former Presidents.
Urine toxicology is negative, and the man appears to be oriented with
respect to person, place, and time. He has had five similar admissions
over the past year. Attempts to interview the patient are fruitless,
as he seems easily derailed from his train of thought. A phone call
to a friend listed in the chart provides the additional information
that the man is homeless, and unable to care for himself. This patient
is exhibiting the signs and symptoms of
A. schizoaffective disorder
B. schizoid personality disorder
CELL BIOLOGY

C. schizophrenia
D. schizophreniform disorder
E. schizotypal personality disorder
Answer
--------------------------------------------------------------------------------

21>Molecular genetic studies are performed on a family with known familial


hypercholesterolemia. In this particular family, the defect in the LDL
receptor gene involves a messenger mutation near the 11th exon, in the
region of homology with epidermal growth factor receptor precursor. A
defect at this site would be most likely to produce which of the following
effects?
A. Decreased transcription of LDL receptor gene
B. Poor internalization of LDL bound to LDL receptor
C. Poor retention of the LDL receptor in the membrane
D. Reduced binding of LDL
E. Trapping of the LDL receptor in the endoplasmic reticulum
Answer
--------------------------------------------------------------------------------

22>A 27-year-old woman is giving birth. During the birth, the placental
membranes tear and amniotic fluid is expressed into a lacerated cervical
vein. Which of the following is the woman most likely to experience
immediately following this event?
A. Hemiplegia
B. Placental abruption
C. Renal failure
D. Respiratory distress
E. Splinter hemorrhages
Answer
--------------------------------------------------------------------------------

23>A 30-year-old pregnant woman complains to her physician of feeling very


tired during her pregnancy. A complete blood count with differential
reveals a hematocrit of 30%, with hypersegmented neutrophils and large,
hypochromic red cells. Deficiency of which of the following would be
most likely to produce these findings?
A. Ascorbic acid
B. Calcium
C. Copper
D. Folate
E. Iron
Answer
--------------------------------------------------------------------------------

24>An otherwise healthy 3-year-old child is brought to the pediatrician with


umbilicated, flesh-colored papules on his trunk. This condition is related
to infection with which of the following viruses?
A. Cytomegalovirus
B. Herpesvirus 6
C. Parvovirus
D. Poxvirus
E. Variola
Answer
--------------------------------------------------------------------------------

25>Which of the following renal structures is most medially located?


CELL BIOLOGY

A. Major calyx
B. Minor calyx
C. Renal cortex
D. Renal pelvis
E. Renal pyramid
Answer
--------------------------------------------------------------------------------

26>A 48-year-old female is being treated for breast carcinoma. Over the past
few days, she has been complaining of dysuria and frequency. Laboratory
examination revealed the presence of microscopic hematuria. The next day
the patient developed gross hematuria. Which of the following agents most
likely caused the development of these signs and symptoms?
A. Cyclophosphamide
B. Mitomycin
C. Paclitaxel
D. Tamoxifen
E. Vincristine
Answer
--------------------------------------------------------------------------------

27>A child who has had abnormal development of the membranous bones has a
broad skull with associated facial and dental anomalies. Which other bones
are most likely to also be affected?
A. Clavicles
B. Femurs
C. Metatarsals
D. Phalanges
E. Tibias
Answer
--------------------------------------------------------------------------------

28>A 3-year-old child is referred to a major medical center because of an


abdominal mass arising from his right adrenal gland. Biopsy of the lesion
demonstrates sheets of small cells with hyperchromatic nuclei containing
occasional pseudorosettes composed of circles of tumor cells with central
young nerve fibers arising from the tumor cells. Which of the following
oncogenes is associated with this patient's tumor?
A. erb-B2
B. c-myc
C. L-myc
D. N-myc
E. ret
Answer
--------------------------------------------------------------------------------

29>A 3-year-old child develops headaches and is brought to the family doctor.
Funduscopic examination reveals papilledema; one retina also shows a very
vascular tumor. CT of the head demonstrates a cystic tumor of the
cerebellum. This child has a high likelihood of later developing
which of the following?
A. Berry aneurysm of the basilar system
B. Bilateral renal cell carcinoma
C. Cancer of a peripheral nerve
D. Choreiform movements related to decreased GABA and acetylcholine
E. Serum cholesterol of greater than 700 mg/dL
Answer
--------------------------------------------------------------------------------
CELL BIOLOGY

30>Administration of an experimental drug that acts on PNS myelin is shown


to increase the space constant of an axon in a peripheral nerve. Action
potentials traveling down the axon would be predicted to be
A. faster
B. larger
C. slower
D. smaller
E. unchanged
Answer
--------------------------------------------------------------------------------

31>A 26-year-old man presents to his physician with a chronic cough. The man
is a smoker, and states that he also gets frequent headaches and aches in
his legs when he exercises. Chest x-ray demonstrates notching of his
ribs. Which of the following undiagnosed congenital defects may be
responsible for these findings?
A. Coarctation of the aorta
B. Eisenmenger's syndrome
C. Tetralogy of Fallot
D. Transposition of great vessels
E. Ventricular septal defect
Answer
--------------------------------------------------------------------------------

32>A 36-year-old Asian male complains of difficulty swallowing.


Esophagoscopy reveals a polypoid mass that is subsequently biopsied.
In addition to tumor cells, the esophageal biopsy show normal smooth
muscle and striated muscle in the same section. Which portion of the
esophagus was the source of this biopsy?
A. Lower esophageal sphincter
B. Lower third of the esophagus
C. Middle third of the esophagus
D. Upper esophageal sphincter
E. Upper third of the esophagus
Answer
--------------------------------------------------------------------------------

33> A 25-year-old woman with sickle cell anemia complains of steady pain in
her right upper quadrant with radiation to the right shoulder, especially
after large or fatty meals. Her physician diagnoses gallstones. Of which
of the following compounds are these stones most likely composed?
A. Calcium bilirubinate
B. Calcium oxalate
C. Cholesterol
D. Cholesterol and calcium bilirubinate
E. Cystine
Answer
--------------------------------------------------------------------------------

34>A 54-year-old alcoholic presents with complaints of tremors and muscle


twitching. Physical examination reveals the presence of Trousseau's sign.
Laboratory data show that serum magnesium is < 1 mEq/L
(normal, 1.4 - 2.2 mEq/L). Which of the following findings
would be most consistent with this information?
A. Decreased serum calcium
B. Decreased serum phosphate
C. Increased bone density
CELL BIOLOGY

D. Increased plasma parathyroid hormone concentration


E. Increased urinary cAMP concentration
Answer
--------------------------------------------------------------------------------

35>A young boy presents with failure to thrive. Biochemical analysis of a


duodenal aspirate after a meal reveals a deficiency of enteropeptidase
(enterokinase). The levels of which of the following digestive enzymes
would be affected?
A. Amylase
B. Colipase
C. Lactase
D. Pepsin
E. Trypsin
Answer
--------------------------------------------------------------------------------

36>A Guatemalan child with a history of meconium ileus is brought to a


clinic because of a chronic cough. The mother notes a history of respiratory
tract infections and bulky, foul-smelling stools. After assessment of the
respiratory tract illness, the physician should also look for signs of
A. cystinuria
B. hypoglycemia
C. iron deficiency anemia
D. sphingomyelin accumulation
E. vitamin A deficiency
Answer
Answers
--------------------------------------------------------------------------------
1>The correct answer is A. A number of physiologic changes occur in a person living at high altitude.
The diminished barometric pressure at high altitude causes alveolar hypoxia and arterial hypoxia.
Pulmonary vasoconstriction occurs in response to alveolar hypoxia; therefore, the diameter of the
pulmonary vessels would be greater in the brother living at sea level. All the other choices describe
physiologic processes that would be enhanced by living at high altitude.
Increased erythropoietin production (choice B), caused by arterial hypoxia, leads to increases in
hematocrit in people living at high altitude.
Mitochondrial density increases (choice C) in people chronically exposed to the hypoxemia caused by
living at high altitude.
At high altitudes, the ventilation rate increases, causing a respiratory alkalosis. The kidney then
compensates by increasing the excretion of HCO3- (choice D).
Increasing the rate of respiration (choice E) is a very useful adaptation to the hypoxic conditions of
high altitude. The primary stimulus is the hypoxic stimulation of peripheral chemoreceptors.
--------------------------------------------------------------------------------
2>The correct answer is A. Alopecia areata is caused by an autoimmune attack on hair follicles. It has
a wide range of clinical severity, with most cases involving a localized patch of hair (which regrows
within 1 year in half of the patients). The hair that does regrow may be gray or depigmented. More
severe cases can involve the entire scalp (alopecia totalis) or, as in this patient, the entire body
surface (alopecia universalis). These more severe cases are less likely to resolve adequately.
Treatment of alopecia areata is often unsuccessful, but topical steroids are typically tried.
Androgenic alopecia (choice B) is common male pattern baldness.
Chronic cutaneous lupus erythematosus (choice C) can produce localized baldness.
Lichen planopilaris (choice D) can produce localized baldness.
Trichotillomania (choice E), also called traumatic alopecia, is alopecia due to trauma, such as hair
pulling or tight braids.
--------------------------------------------------------------------------------3>The correct answer is B.
CELL BIOLOGY

Both the omental bursa and the greater omentum are derived from the dorsal mesogastrium, which is
the mesentery of the stomach region.
The dorsal mesoduodenum (choice A) is the mesentery of the developing duodenum, which later
disappears so that the duodenum and pancreas come to lie retroperitoneally.
The pericardioperitoneal canal (choice C) embryologically connects the thoracic and peritoneal canals.
The pleuropericardial membranes (choice D) become the pericardium and contribute to the
diaphragm.
The ventral mesentery (choice E) forms the falciform ligament, ligamentum teres, and lesser
omentum.
--------------------------------------------------------------------------------4>The correct answer is C.
Wine and cheese (and many other fermented foods) contain tyramine, an indirect sympathomimetic
that can trigger excess catecholamine release and lead to a hypertensive crisis when ingested by
patients taking MAO inhibitors. The only such drug listed among the answer choices is phenelzine.
Other MAO inhibitors with similar effects include tranylcypromine, isocarboxazid, and iproniazid.
Whenever this particular drug class is mentioned in a question stem, consider the possibility of
interactions with foods or other medications the patient may have taken.
Amitriptyline (choice A) is a tricyclic antidepressant. Tricyclic antidepressant drugs (particularly
amitriptyline) are known for their anticholinergic side effects. They also produce postural hypotension
(because they block alpha-adrenergic receptors) and are sedative.
Fluoxetine and sertraline (choices B and D) are antidepressants that are selective serotonin reuptake
inhibitors (SSRIs). Fluoxetine is also useful in treatment of obsessive-compulsive disorders.
Trazodone (choice E) is an atypical antidepressant with substantial sedative side effects. Its most
serious side effect is priapism, a medical emergency.
--------------------------------------------------------------------------------5>The correct answer is D.
The cells of the anterior pituitary can be classified as chromophils (love dyes) or chromophobes (do
not stain with dyes). The chromophils can be further divided into acidophils (stain with acidic dyes)
and basophils (stain with basic dyes). The acidophils include the somatotropes, which secrete growth
hormone, and the mammotropes, which secrete prolactin. The basophils include the corticotropes,
which secrete ACTH (choice A), the gonadotropes, which secrete FSH and LH (choices B and C), and
the thyrotropes, which secrete TSH (choice E).
--------------------------------------------------------------------------------6>The correct answer is D.
Persons with this condition often perceive attacks and danger in relatively innocuous situations. They
are quick to respond with anger, and, because personality disorders are ego-syntonic, individuals with
personality disorders do not believe themselves to be in error.
The individual with dependent personality disorder (choice A) does not confront others but wants
others to take care of him.
The individual with histrionic personality disorder (choice B) is flamboyant and seductive, not
confrontational and angry.
The individual with narcissistic personality disorder (choice C) is characterized by feelings of
entitlement because they are so "special."
And the individual with passive aggressive personality disorder (choice E) expresses anger indirectly
(e.g., always being late) rather than confronting
--------------------------------------------------------------------------------7>The correct answer is C.
The loading of O2 is facilitated when the oxygen dissociation curve shifts to the left, and the unloading
of O2 is facilitated when the oxygen dissociation curve shifts to the right. A good way to remember the
conditions that promote dissociation of O2 is to think of exercising muscle, which has decreased pH
(choice C) because of the accumulation of lactic acid, increased PCO2 (compare with choice B)
because of the increased rate of aerobic metabolism, increased temperature (compare with choice D),
and increased 2,3-DPG (2,3-diphosphoglycerate; compare with choice E) because of increased
glycolysis.
Carbon monoxide poisoning (choice A) left-shifts the oxygen dissociation curve, which interferes with
CELL BIOLOGY

the unloading of O2. Carbon monoxide also strongly binds to available sites on hemoglobin.
--------------------------------------------------------------------------------8>The correct answer is A.
This patient has Huntington's disease, which has autosomal dominant inheritance. It is characterized
by severe degeneration of the caudate nucleus along with degenerative changes in the putamen and
cortex. In addition to chorea, these patients frequently suffer from athetoid (writhing) movements,
progressive dementia, and behavioral disorders.
--------------------------------------------------------------------------------9>The correct answer is E.
The rash described is that of secondary syphilis, caused by Treponema pallidum. Involvement of
palms and soles by a rash is unusual, and secondary syphilis should come to mind. Not all patients
with secondary syphilis have a severe form of the rash, and consequentially some cases are missed.
Primary syphilis takes the form of a painless, button-like mass called chancres. Tertiary syphilis, which
is now rare, has a propensity for involving the aorta and central nervous system and can also cause
"gummas" (granulomatous-like lesions) in many sites, notably including liver and bone.
Herpes simplex I (choice A) usually causes perioral vesicular lesions.
Herpes simplex II (choice B) usually causes genital vesicular lesions.
HIV (choice C) does not itself cause a rash, although co-infection with other organisms can result in a
rash.
Neisseria gonorrhoeae (choice D) does not typically cause a rash.
--------------------------------------------------------------------------------10>The correct answer is B.
Hyperlipidemia has been subclassified based on the lipid and lipoprotein profiles. Type 2a, which this
patient has, can be seen in a hereditary form, known as familial hypercholesterolemia, and also in
secondary, acquired forms related to nephritic syndrome and hyperthyroidism. The root problem
appears to be a deficiency of LDL receptors, which leads to a specific elevation of cholesterol in the
form of increased LDL. Heterozygotes for the hereditary form generally develop cardiovascular disease
from 30 to 50 years of age. Homozygotes may have cardiovascular disease in childhood.
Type 1 (choice A) is characterized by isolated elevation of chylomicrons.
Type 2b (choice C) is characterized by elevations of both cholesterol and triglycerides in the form of
LDL and VLDL.
Type 3 (choice D) is characterized by elevations of triglycerides and cholesterol in the form of
chylomicron remnants and IDL.
Type 5 (choice E) is characterized by elevations of triglycerides and cholesterol in the form of VLDL
and chylomicrons.
--------------------------------------------------------------------------------11The correct answer is C. The
neurohypophysis (posterior pituitary) is derived from an evagination of diencephalic neurectoderm.
This structure is responsible for releasing oxytocin and vasopressin to the general circulation. Both
hormones are synthesized in cell bodies contained within the hypothalamus.
ACTH (choice A), prolactin (choice D), and TSH (choice E) are all synthesized and released by the
anterior pituitary, or adenohypophysis, which is derived from an evagination of the ectoderm of
Rathke's pouch, a diverticulum of the primitive mouth. Remnants of this pouch may give rise to a
craniopharyngioma in later life.
Epinephrine (choice B) is synthesized and released into the circulation by the adrenal medulla, a
neural crest derivative.
--------------------------------------------------------------------------------12>The correct answer is E.
The most common bacteria implicated in community-acquired pneumonia is the pneumococcus,
Streptococcus pneumoniae. Other organisms frequently implicated in patients less than age 60
without comorbidity include Mycoplasma pneumoniae, respiratory viruses, Chlamydia pneumoniae,
and Haemophilus influenzae. When community-acquired pneumonia occurs in elderly patients or
patients with comorbidity, aerobic gram-negative bacilli and Staphylococcus aureus are added to the
list.
The organisms listed in choices A, B, and C are important causes of community-acquired pneumonia,
but are not the most frequent causes.
CELL BIOLOGY

Staphylococcus aureus (choice D) is an important cause of community-acquired pneumonia


(particularly in the elderly and in patients with comorbidity), but is not the most frequent cause.
--------------------------------------------------------------------------------13>The correct answer is E.
Uric acid kidney stones in patients with leukemia are secondary to increased production of uric acid
from purine breakdown during periods of active cell proliferation, especially following treatment.
Vigorous hydration and diuresis are generally instituted after the diagnosis of acute leukemia is made.
Uric acid kidney stones are also associated with inborn errors of purine metabolism, such as gout.
Pigment gallstones (choice A) are associated with hemolytic disease. The incidence of this type of
gallstone is not increased in treated leukemias.
Cholesterol gallstones (choice B) are associated with diabetes mellitus, obesity, pregnancy, birth
control pills, and celiac disease.
Cystine kidney stones (choice C) are rare; they are found in cystinuria.
Struvite kidney stones (choice D) are associated with infection by urea-splitting organisms, such as
Proteus.
--------------------------------------------------------------------------------14>The correct answer is B.
The data shown in the table indicate that the man has developed metabolic alkalosis (increased PCO2,
pH, and HCO3-), which occurs commonly with overuse of diuretics (thiazides and loop diuretics). The
overuse of a loop diuretic increases the excretion of sodium (choice E) and potassium (choice C) by
the kidneys. The increase in potassium excretion leads to a decrease in plasma potassium levels
(choice B). The decrease in plasma potassium stimulates aldosterone secretion, which raises plasma
aldosterone levels (choice A). The sodium depletion stimulates renin secretion (choice D), which in
turn raises angiotensin II levels in the plasma (which also stimulates aldosterone secretion
--------------------------------------------------------------------------------15>The correct answer is C.
This is a case of Reiter's syndrome. Patients typically present with the acute onset of arthritis (usually
asymmetric and additive), with involvement of new joints occurring over a period of a few days to 2
weeks. Joints of the lower extremities are the most commonly involved, but wrists and fingers can
also be affected. Dactylitis (sausage digit), a diffuse swelling of a solitary finger or toe, is a distinctive
feature of Reiter's arthritis and psoriatic arthritis. Tendonitis and fasciitis are common. Spinal pain and
low back pain are common. Conjunctivitis, urethritis, diarrhea, and skin lesions are also associated
with Reiter's syndrome. Up to 75% of patients are HLA-B27 positive. Microorganisms which can trigger
Reiter's syndrome include Shigella spp., Salmonella spp., Yersinia spp., Campylobacter jejuni, and
Chlamydia trachomatis. Most patients are younger males.
Gout (choice A) usually presents as an explosive attack of acute, very painful, monarticular
inflammatory arthritis. Hyperuricemia is the cardinal feature and prerequisite for gout. The first
metatarsophalangeal joint is involved in over 50% of first attacks.
Lyme disease (choice B), caused by Borrelia burgdorferi, presents with a red macule or papule at the
site of the tick bite. This lesion, called erythema chronicum migrans, slowly expands to form a large
annular lesion with a red border and central clearing. The lesion is warm, but usually not painful. The
patient also has severe headache, stiff neck, chills, arthralgias, and profound malaise and fatigue.
Untreated infection is associated with development of arthritis. The large joints (e.g., knees) are
usually involved with the arthritis lasting for weeks to months.
Rheumatoid arthritis (choice D) begins insidiously with fatigue, anorexia, generalized weakness, and
vague musculoskeletal symptoms leading up to the appearance of synovitis. Pain in the affected
joints, aggravated by movement, is the most common manifestation of established rheumatoid
arthritis. Generalized stiffness is frequent and is usually greatest after periods of inactivity. Morning
stiffness of greater than 1 hour in duration is very characteristic. Rheumatoid arthritis is more
common in females. The metacarpophalangeal and proximal interphalangeal joints of the hands are
characteristically involved.
Septic arthritis (choice E) is caused by a variety of microorganisms, including Neisseria gonorrhoeae
and Staphylococcus aureus. Hematogenous spread is the most common route in all age groups. 90%
of patients present with involvement of a single joint, usually the knee. The usual presentation is
CELL BIOLOGY

moderate-to-severe pain, effusion, muscle spasm, and decreased range of motion. Peripheral
leukocytosis and a left shift are common. Disseminated gonococcal infections present as fever, chills,
rash, and articular symptoms. Papules progressing to hemorrhagic pustules develop on the trunk and
extensor surfaces of the distal extremities. Migratory arthritis and tenosynovitis of multiple joints is
common.
--------------------------------------------------------------------------------16>The correct answer is D.
Many people who request physician-assisted suicide have one of two conditions present: either a
poorly controlled painful condition or severe depression. If the painful condition is adequately treated
or the depression is brought under good medical control, the request for physician assistance in
terminating the situation is typically withdrawn. It is important to note that bringing these conditions
under control requires the intervention of caregivers who are specifically trained in the management of
these two conditions; primary care physicians usually are not adequately trained to address these
difficult presentations.
While patients who are diagnosed as bipolar disorder (choice A), borderline personality disorder
(choice B), and schizophrenic disorder (choice E) often make suicide attempts (and frequently
complete those attempts), they do not generally ask their physician for assistance in the suicide.
Persons with factitious disorder (choice C) are seeking primary gain, often for dependency needs, and
are seeking to enter the "sick role" not the "dead role."
--------------------------------------------------------------------------------17>The correct answer is E.
The patient has bilateral acoustic neuromas, probably due to neurofibromatosis type II (over 90% of
patients with NF-2 develop bilateral acoustic neuromas). This condition is a associated with the NF-2,
gene, located on 22q (note all the 2's). Patients often develop meningiomas, gliomas, and
schwannomas of cranial and spinal nerves.
5q (choice A) contains the APC tumor suppressor gene, which is associated with familial and sporadic
colorectal cancers.
13q (choice B) contains the Rb tumor suppressor gene, which is associated with retinoblastoma and
osteosarcoma.
17q (choice C) contains both the NF-1 tumor suppressor gene, which is associated with
neurofibromatosis type I, and the p53 tumor suppressor gene, associated with many human cancers.
18q (choice D) contains both the DCC gene, which is associated with colon and gastric carcinomas and
the DPC gene, associated with pancreatic cancer.

--------------------------------------------------------------------------------18>
The correct answer is D. Perforation of a peptic ulcer is potentially fatal, because of either peritonitis
with sepsis or sudden exsanguination (if the perforation damages one of the many arteries of the
stomach). Peptic ulcer disease, gastritis, and possibly gastric carcinoma and gastric lymphoma have
been strongly associated with Helicobacter pylori colonization of the mucus layer covering the gastric
mucosa. Colonization is associated with destruction of the mucus layer, thereby destroying its
protective function.
Cryptosporidium parvum (choice A) causes diarrhea that is severe in immunocompromised patients.
Entamoeba histolytica (choice B) produces dysentery-like symptoms or can cause liver abscess.
Escherichia coli (choice C) causes a variety of diarrheal diseases and can infect the bladder and soft
tissues.
Mycobacterium tuberculosis (choice E) causes tuberculosis, characterized by granuloma formation,
especially in the lungs.
--------------------------------------------------------------------------------19>The correct answer is E.
By the third week of development, hematopoiesis begins in the blood islands of the yolk sac.
Beginning at 1 month of age and continuing until 7 months of age, blood elements are also formed in
the liver. Hematopoiesis occurs in the spleen and lymphatic organs between 2 and 4 months, and in
the bone marrow after 4 months.
--------------------------------------------------------------------------------
CELL BIOLOGY

15>The correct answer is C. The patient is suffering from schizophrenia. The key to the diagnosis of
psychosis is that there has been a marked decline in the level of functioning (i.e., the man is homeless
and cannot care for himself). Although hallucinations or delusions are not mentioned in the case
history, the presence of disorganized speech, grossly disorganized behavior, and the duration of
symptoms (longer than six months) suggest a diagnosis of schizophrenia.
In schizoaffective disorder (choice A), alterations in mood are present during a substantial portion of
the illness.
Although schizoid personality disorder (choice B) produces detachment from social relationships and is
characterized by restriction of emotional expression, it is not accompanied by a marked decline in
occupational functioning.
Schizophreniform disorder (choice D) is characterized by schizophrenic-like symptoms, but the
duration of symptoms is, by definition, less than six months.
Schizotypal personality disorder (choice E) is characterized by eccentricities of behavior, odd beliefs or
magical thinking, and difficulties with social and interpersonal relationships. Unlike schizophrenia,
schizotypal personality disorder is not characterized by a formal thought disorder.
--------------------------------------------------------------------------------

--------------------------------------------------------------------------------21>The correct answer is E.


Familial hypercholesterolemia, which is due to defective function of the LDL receptor, is an area of
intense research. The molecular basis of LDL receptor abnormalities is becoming better understood,
and more than 200 mutations in the gene for the LDL receptor have been identified. The gene has 5
general domains and 18 exons. Defects near exons 7 to 14 (including this case) are in the region of
homology with epidermal growth factor receptor precursor. This region of the molecule is needed for
dissociation of LDL from the receptor in the endosome. Receptors with a defect in this area
(sometimes called class II mutations) also have trouble being initially transported to the Golgi
complex (transport-deficiency alleles) and become trapped in endoplasmic reticulum.
Decreased transcription of the LDL receptor gene (choice A) is considered a class I mutation and
involves the signal sequence domain near exon 1.
Poor internalization of LDL bound to LDL receptor (choice B) is considered a class IV mutation. Such
mutations are associated with the membrane-spanning/cytoplasmic domain, specifically near exon
18.
Poor retention of the LDL receptor in the membrane (choice C) is considered a class IV mutation and
is associated with the membrane-spanning/cytoplasmic domain, specifically near exons 2-6.
Reduced binding of LDL (choice D) is considered a class III mutation and involves the LDL binding
domain near exons 2-6.
--------------------------------------------------------------------------------22>The correct answer is D.
Respiratory distress immediately follows amniotic fluid embolism as the emboli consisting of squamous
cells, lanugo, and mucus deposit in the pulmonary microcirculation, producing numerous tiny
pulmonary infarcts. The dramatic respiratory distress may also reflect the action of prostaglandins and
other bioactive compounds present in high concentrations in the amniotic fluid embolus.
Hemiplegia (choice A) would reflect an ischemic injury to one hemisphere of the cerebrum or the
brainstem. A venous embolus would not produce such an insult.
Placental abruption (choice B) is partial, premature separation of the placental disc from the
endometrium. Although abruption may occur in this setting, it is not a result of an amniotic fluid
embolism.
There are numerous causes of renal failure (choice C); the most likely ones in the peripartum interval
include eclampsia, hypovolemic shock, and ascending infections. Amniotic fluid embolism would be
expected to produce severe dyspnea well before shock and renal failure might arise.
Splinter hemorrhages (choice E) are small hemorrhages seen on toes and fingers due to a shower of
microemboli arising in the arterial circulation. Amniotic fluid emboli arise in the veins and deposit in
CELL BIOLOGY

the lungs.
--------------------------------------------------------------------------------23>The correct answer is D.
The patient has a megaloblastic anemia, which can be due to deficiency of folate or B12. Pregnancy
increases the need for folate and other nutrients used by both baby and mother, and may "unmask" a
borderline dietary deficiency. For this reason, most obstetricians recommend vitamin supplements for
pregnant women.
Ascorbic acid (choice A) is vitamin C, and its deficiency predisposes for capillary fragility and oral
lesions.
Calcium deficiency (choice B) predisposes for osteoporosis/osteopenia.
Copper deficiency (choice C) is rare; when it occurs, it may cause a hypochromic anemia, neutropenia,
osteoporosis, or hypotonia.
Iron deficiency (choice E) causes a microcytic, hypochromic anemia, with reduced mental and physical
performance.
--------------------------------------------------------------------------------24>The correct answer is D.
The lesions are characteristic of molluscum contagiosum, which is a typically benign and self-limited
condition caused by a poxvirus. The disease can be transmitted either venereally or through non-
venereal contact. The other viruses listed do not cause similar skin lesions. Patients with advanced
HIV infection may develop a severe, generalized, and persistent eruption, often involving the face and
upper body.
Cytomegalovirus (choice A) causes congenital infections and disseminated infections in
immunosuppressed patients.
Herpesvirus 6 (choice B)causes roseola (exanthem subitum).
Parvovirus (choice C) causes aplastic crises in patients with hemolytic anemia.
Variola (choice E) is the smallpox virus.
--------------------------------------------------------------------------------25>The correct answer is D.
This is a relatively simple question that requires you to visualize the relationship among the key parts
of the kidney and to identify the one that lies most medially. If you think about it for a second, since
the kidneys ultimately drain into the ureter at their medial poles, you are looking for the structure that
is closest to the ureter. The correct answer is the renal pelvis. The renal pelvis is the dilated upper
portion of the ureter that receives the major calyces.
In terms of the other answer choices, the order from most lateral to most medial is: renal cortex
(choice C), renal pyramid (choice E), minor calyx (choice B), major calyx (choice A), and then the
renal pelvis (choice D).
--------------------------------------------------------------------------------
26>
The correct answer is A. Cyclophosphamide is metabolized to acrolein, which is excreted in the urine.
If the patient's urine is concentrated, the toxic metabolite may cause severe bladder damage. Early
symptoms of bladder toxicity include dysuria and frequency. This can be distinguished from a urinary
tract infection, since there is no bacteriuria with cyclophosphamide-induced bladder toxicity. However,
microscopic hematuria is often present on urinalysis. In severe hemorrhagic cystitis, large segments of
the bladder mucosa may be shed which can lead to prolonged, gross hematuria. The incidence of
cyclophosphamide-induced hemorrhagic cystitis can be decreased by ensuring that the patient
maintains a high fluid intake. Cyclophosphamide is an alkylating agent used in the treatment of breast
carcinoma, malignant lymphoma, multiple myeloma, and adenocarcinoma of the ovary, as well as
various other forms of cancer. The major toxic reactions commonly seen with this agent include
mucositis, nausea, hepatotoxicity, sterile hemorrhagic and non-hemorrhagic cystitis, leukopenia,
neutropenia, and interstitial pulmonary fibrosis.
Mitomycin (choice B) is an antibiotic antineoplastic agent used in the treatment of breast carcinoma,
adenocarcinoma of the pancreas and stomach, as well as various other forms of cancer. The major
toxic reactions commonly seen with this agent include bone marrow depression, nausea,
hepatotoxicity, acute bronchospasm, thrombocytopenia, and interstitial pneumonitis.
CELL BIOLOGY

Paclitaxel (choice C) is an antineoplastic agent primarily used in the treatment of ovarian and breast
cancer. The major toxic reactions commonly seen with this agent include bone marrow depression,
nausea, hepatotoxicity, bronchospasm, thrombocytopenia, and neutropenia.
Tamoxifen (choice D) is an antineoplastic hormone primarily used in the palliative treatment of
estrogen-receptor positive breast cancer patients. The major toxic reactions commonly seen with this
agent include depression, dizziness, thrombosis, mild leukopenia or thrombocytopenia.
Vincristine (choice E) is a mitotic inhibitor antineoplastic agent used in the treatment of breast cancer,
Hodgkin's disease, non-Hodgkin's lymphoma, advanced testicular cancer and various other types of
cancer. The major toxic reactions commonly seen with this agent include mental depression,
hemorrhagic enterocolitis, bone marrow depression, nausea, thrombocytopenia, and leukopenia
--------------------------------------------------------------------------------27>The correct answer is A.
In a syndrome called cleidocranial dysostosis, absence of part of the clavicles accompanies a broad
skull, and facial and dental anomalies. Note that you could also have answered this question by noting
that of the bones listed, only the clavicles form by intramembranous ossification.
The femurs (choice B), metatarsals (choice C), phalanges (choice D), and tibias (choice E) are
cartilaginous (formed by endochondral ossification) rather than membranous bones.
--------------------------------------------------------------------------------28>The correct answer is D.
The tumor is a neuroblastoma, which is one of the principal forms of cancer in children.
Neuroblastoma typically occurs before age 5, with many presenting before age 2. Neuroblastoma can
arise from neural crest cells throughout the body, but the adrenal medulla is the most common site.
Homer-Wright pseudorosettes are circles of tumor cells with central young nerve fibers arising from
the tumor cells. The oncogene associated with neuroblastoma is N-myc.
erb-B2 (choice A) is associated with breast, ovarian, and gastric carcinomas.
c-myc (choice B) is associated with Burkitt's lymphoma.
L-myc (choice C) is associated with small cell carcinoma of the lung.
ret (choice E) is associated with multiple endocrine neoplasia, types II and III.
--------------------------------------------------------------------------------29>The correct answer is B.
The disease is von Hippel-Landau disease, which is associated with a deletion involving the VHL gene
on chromosome 3 (3p). Affected individuals develop vascular tumors (hemangioblastomas) of the
retina, cerebellum, and/or medulla. Roughly half of the affected individuals later develop multiple,
bilateral renal cell carcinomas.
Berry aneurysms (choice A) are unrelated to hemangioblastomas, but are instead associated with
adult polycystic disease.
Peripheral nerve cancers (choice C) are a feature of von Recklinghausen's disease (neurofibromatosis
type I).
Choreiform movements, related to decreased GABA and acetylcholine (choice D), are a feature of
Huntington's disease.
Extremely high serum cholesterol (choice E) suggests the homozygous form of familial
hypercholesterolemia.
--------------------------------------------------------------------------------30>The correct answer is A.
The space constant of an axon reflects the amount of passive or electrotonic spread of current within
an axon. The larger the space constant, the further the current can spread, allowing action potentials
to propagate faster. This is why myelin increases the conduction velocity of action potentials down an
axon. Conversely, demyelination decreases the space constant and slows action potential conduction.
--------------------------------------------------------------------------------31>The correct answer is A.
Coarctation of the aorta occurs in two patterns. In the infantile type, the stenosis is proximal to the
insertion of the ductus arteriosus (preductal); this pattern is associated with Turner's syndrome. In
the adult form, the stenosis is distal to the ductus arteriosus (postductal) and is associated with
notching of the ribs (secondary to continued pressure from the aorta on them), hypertension in the
upper extremities, and weak pulses in the lower extremities. Headache, cold extremities, and lower
extremity claudication with exercise are typical if the patient is symptomatic (many adults with mild
CELL BIOLOGY

distal coarctation may remain asymptomatic for years). Upper extremity hypertension with weak
pulses in the lower extremities, and a midsystolic (or continuous) murmur over the chest or back may
be the only obvious signs in some. Note that the chronic cough is probably related to the man's
smoking, and is not caused by the coarctation.
Eisenmenger's syndrome (choice B) is a shift from a left-to-right shunt to a right-to-left shunt
secondary to pulmonary hypertension.
Tetralogy of Fallot (choice C) and transposition of great arteries (choice D) cause cyanosis and are
usually diagnosed in infancy.
Ventricular septal defect (choice E) might remain undiagnosed until adulthood, but would not cause
notching of the ribs.
--------------------------------------------------------------------------------32>The correct answer is C.
The muscularis of the upper third of the esophagus (choice E) is composed entirely of striated muscle.
The middle third (choice C) contains both striated and smooth muscle. The lower third (choice B) and
lower esophageal sphincter (choice A) contain only smooth muscle. There is no such thing as the
upper esophageal sphincter (choice D).
--------------------------------------------------------------------------------33>The correct answer is A.
Bilirubin is a degradative product of hemoglobin metabolism. Bilirubin (pigment) stones are specifically
associated with excessive bilirubin production in hemolytic anemias, including sickle cell anemia.
Bilirubin stones can also be seen in hepatic cirrhosis and liver fluke infestation.
Calcium oxalate stones (choice B) and cystine stones (choice E) are found in the kidney, rather than
the gallbladder.
Pure cholesterol stones (choice C) are less common than mixed gallstones, but have the same risk
factors, including obesity and multiple pregnancies.
Mixed stones (choice D) are the common "garden variety" gallstones, found especially in obese,
middle aged patients, with a female predominance
--------------------------------------------------------------------------------
34>
The correct answer is A. Malnutrition associated with chronic alcoholism can lead to a severe
magnesium deficiency. The effect of low serum magnesium on parathyroid hormone secretion (PTH)
depends on severity and duration. An acute decrease in serum magnesium will increase PTH secretion,
while a prolonged severe deficiency results in decreased PTH secretion. There is also evidence that the
action of PTH is decreased with chronic magnesium deficiency. Hence, this patient is suffering from
"functional" hypoparathyroidism. The low serum calcium can produce weakness, tremors, muscle
fasciculations, and seizures. A positive Trousseau's sign indicates the presence of latent tetany. It is
observed by inflating a blood pressure cuff above systolic blood pressure for at least 2 minutes. A
positive reaction consists of the development of carpal spasm, with relaxation occurring within
seconds after deflating the cuff. In patients with magnesium deficiency, magnesium administration will
produce a prompt rise in plasma PTH with subsequent restoration of serum calcium concentration to
normal.
With functional hypoparathyroidism bone density would be decreased (not increased, choice C).
The combination of decreased PTH secretion (not increased, choice D) and decreased effectiveness of
PTH produce hypocalcemia and hyperphosphatemia (not hypophosphatemia, choice B).
Urinary cAMP would probably be decreased (not increased, choice E), given the low PTH.
--------------------------------------------------------------------------------35>The correct answer is E.
Enteropeptidase, formerly called enterokinase, activates trypsinogen by limited proteolytic digestion to
give trypsin. Trypsin is itself capable of activating trypsinogen, which produces a positive feedback
effect. Trypsin also activates chymotrypsinogen (and several other proteolytic enzymes), so deficiency
of enteropeptidase results in a severe deficiency of enzymes that digest protein.
Amylase (choice A) aids in the breakdown of starches to oligosaccharides, maltose, and maltotriose.
Colipase (choice B), along with other lipases, functions to digest fats.
Lactase (choice C) is a brush-border disaccharidase that hydrolyzes the bond between galactose and
CELL BIOLOGY

glucose in lactose.
Pepsin (choice D) is a proteolytic enzyme secreted in an inactive form (pepsinogen) by the chief cells
of the stomach. Pepsinogen is activated by stomach acid, and so is not dependent on enteropeptidase.
Pepsin alone will not replace the activities of other proteolytic enzymes, partly because food does not
remain in the stomach for an extended period of time.
--------------------------------------------------------------------------------36>The correct answer is E.
The child is likely suffering from cystic fibrosis. In this disorder, an abnormality of chloride channels
causes all exocrine secretions to be much thicker, and more viscous than normal. Pancreatic secretion
of digestive enzymes is often severely impaired, with consequent steatorrhea and deficiency of fat-
soluble vitamins, including vitamin A.
Cystinuria (choice A) is a relatively common disorder in which a defective transporter for dibasic amino
acids (cystine, ornithine, lysine, arginine; COLA) leads to saturation of the urine with cystine, which is
not very soluble in urine, and precipitates out to form stones.
Hypoglycemia (choice B) is not a prominent feature of children with cystic fibrosis who are on a
normal diet. Hyperglycemia may occur late in the course of the disease.
Iron deficiency anemia (choice C) is not found with any regularity in children with cystic fibrosis.
Sphingomyelin accumulation (choice D) is generally associated with deficiency of sphingomyelinase, as
seen in Niemann-Pick disease.
--------------

USMLE STEP1 MCQ STUDY ONLINE QUESTIONS REVIEW


1> A normal, healthy, 25-year-old man lives at the beach. His twin brother has
been living in a mountain cabin for the past 2 years. Which of the
following indices would be expected to be higher in the man living at
sea level?
A. Diameter of pulmonary vessels
B. Erythropoietin production
C. Mitochondrial density in a muscle biopsy
D. Renal bicarbonate (HCO3-) excretion
E. Respiratory rate
Answer
--------------------------------------------------------------------------------
2>A woman presents to a dermatologist because she has lost almost all the
hair on her body, including scalp hair, eye brows, eye lashes, arm pit and
groin hair, and the fine hairs on her body and extremities. She most likely
has a variant of which of the following?
A. Alopecia areata
B. Androgenic alopecia
C. Chronic cutaneous lupus erythematosus
D. Lichen planopilaris
E. Trichotillomania
Answer
--------------------------------------------------------------------------------

3>The greater omentum is derived from which of the following embryonic


structures?
A. Dorsal mesoduodenum
B. Dorsal mesogastrium
C. Pericardioperitoneal canal
D. Pleuropericardial membranes
E. Ventral mesentery
Answer
-----------------------------------------------------------------------------
CELL BIOLOGY

4>A 34-year-old woman is prescribed an antidepressant, which she has taken


for the past 3 months. She is on no other medications and is in generally
good health. After attending a party, at which she consumed wine and
cheese, she is rushed to the emergency room with tachycardia, headache,
and a blood pressure of 200/100. Which antidepressant is she most likely
taking?
A. Amitriptyline
B. Fluoxetine
C. Phenelzine
D. Sertraline
E. Trazodone
Answer
--------------------------------------------------------------------------------

5>Which of the following hormones is secreted by anterior pituitary cells


that stain with acidic dyes?
A. ACTH
B. FSH
C. LH
D. Prolactin
E. TSH

6>A 23-year-old male graduate student gets into a serious argument with
one of his college professors, making a physical threat to the professor,
and necessitating a call to campus security. The argument was precipitated
by an incident between the professor and the student's girlfriend; when
the professor corrected the student's girlfriend in class, the student
felt the professor was verbally abusive. Which of the following is
the most likely diagnosis ?
A. Dependent personality disorder
B. Histrionic personality disorder
C. Narcissistic personality disorder
D. Paranoid personality disorder
E. Passive aggressive personality disorder
Answer
--------------------------------------------------------------------------------

7>Which of the following would shift the oxygen-hemoglobin dissociation curve


to the right?
A. Carbon monoxide poisoning
B. Decreased PCO2
C. Decreased pH
D. Decreased temperature
E. Decreased 2,3-DPG
Answer
--------------------------------------------------------------------------------

8>The wife of a 48-year-old male patient brings him to the emergency room and
says that his memory has progressively gotten worse over the last several
years. She also says his personality has been changing. The physician notes
abnormal writhing movements of the man's limbs and hyperreactive reflexes
. MRI reveals a loss of volume in the neostriatum and cortex. This disease
is inherited via an
CELL BIOLOGY

A. autosomal dominant trait


B. autosomal recessive trait
C. x-linked dominant trait
D. x-linked recessive trait
Answer
--------------------------------------------------------------------------------

9>A man presents to a dermatologist because of a severe mucocutaneous rash


that involves most of his body, including his palms and soles. Questioning
reveals that he is a merchant marine who several months previously had an
encounter with a prostitute in Southeast Asia. Which of the following is
the most likely causative agent of this rash?
A. Herpes simplex I
B. Herpes simplex II
C. HIV
D. Neisseria gonorrhoeae
E. Treponema pallidum
Answer
--------------------------------------------------------------------------------

10>A patient with familial hypercholesterolemia undergoes a detailed serum


lipid and lipoprotein analysis. Studies demonstrate elevated cholesterol
in the form of increased LDL without elevation of other lipids.
This patient's hyperlipidemia is best classified as which of the
following types?
A. Type 1
B. Type 2a
C. Type 2b
D. Type 3
E. Type 5
Answer
--------------------------------------------------------------------------------

11>A mutation affecting the development of the diencephalon could interfere


with the secretion of which of the following hormones?
A. Adrenocorticotrophic hormone (ACTH)
B. Epinephrine
C. Oxytocin
D. Prolactin
E. Thyroid stimulating hormone (TSH)
Answer
--------------------------------------------------------------------------------

12>Which of the following organisms is the most common cause of


community-acquired pneumonia?
A. Chlamydia pneumoniae
B. Haemophilus influenzae
C. Mycoplasma pneumoniae
D. Staphylococcus aureus
E. Streptococcus pneumoniae
Answer
--------------------------------------------------------------------------------

13>A 54-year-old male with acute lymphocytic leukemia develops a blast crisis.
CELL BIOLOGY

He is treated with intensive systemic chemotherapy. Following treatment,


the patient will be at increased risk for the development of
A. bile pigment gallstones
B. cholesterol gallstones
C. cystine kidney stones
D. struvite kidney stones
E. uric acid kidney stones
Answer
--------------------------------------------------------------------------------

14>A 65-year-old man with mild heart failure is treated with a loop diuretic
. A few days later the man complains of muscle weakness. Laboratory results
are shown below. Arterial PCO2: 48 mm Hg Arterial pH: 7.49
Plasma HCO3-: 35 mEq/L Which of the following is most likely
decreased in this man?
A. Plasma aldosterone
B. Plasma potassium
C. Potassium excretion
D. Renin secretion
E. Sodium excretion
Answer
--------------------------------------------------------------------------------

15>A 27-year-old white male presents with a 3-week history of several swollen
and painful toes and knees. He has a past history of conjunctivitis. He
also describes some low back stiffness that is more severe in the morning.
Which of the following is the most likely diagnosis?
A. Gout
B. Lyme disease
C. Reiter's syndrome
D. Rheumatoid arthritis
E. Septic arthritis
Answer
--------------------------------------------------------------------------------

16>A 37-year-old woman who was diagnosed with AIDS 3 years earlier is unable
to work, is physically debilitated, and requests her physician to provide
her with medications with which to take her own life. The most common
emotional disorder that results in such requests by patients is
A. bipolar I disorder, manic type
B. borderline personality disorder
C. factitious disorder
D. major depressive disorder
E. schizophrenic disorder
Answer
--------------------------------------------------------------------------------
17>
A 25-year-old man presents with a chief complaint of persistent,
high-pitched ringing noises in his ears. Questioning reveals that
he has also been losing his balance lately. CT of the head demonstrates
bilateral tumors involving the vestibulocochlear nerve. Which of the
following chromosomes contains the tumor suppressor gene most likely to
be involved in this case?
A. 5q
CELL BIOLOGY

B. 13q
C. 17q
D. 18q
E. 22q
Answer
--------------------------------------------------------------------------------

18> An autopsy is performed on a man who suddenly began vomiting voluminous


quantities of blood and exsanguinated. Which of the following organisms is most
likely implicated in the pathogenesis of this disease?
A. Cryptosporidium parvum
B. Entamoeba histolytica
C. Escherichia coli
D. Helicobacter pylori
E. Mycobacterium tuberculosis
Answer
--------------------------------------------------------------------------------

19>During embryological development, hematopoiesis occurs in different organs


at different times. Which of the following are the correct organs, in the
correct sequence, at which hematopoiesis occurs embryologically?
A. Amnion, yolk sac, placenta, bone marrow
B. Placenta, liver and spleen, yolk sac, bone marrow
C. Placenta, spleen and lymphatic organs, bone marrow
D. Yolk sac, bone marrow, liver and spleen
E. Yolk sac, liver, spleen and lymphatic organs, bone marrow
Answer
--------------------------------------------------------------------------------

20>A 27-year-old male is brought into the emergency room by the police,
who found him walking aimlessly, shouting the names of former Presidents.
Urine toxicology is negative, and the man appears to be oriented with
respect to person, place, and time. He has had five similar admissions
over the past year. Attempts to interview the patient are fruitless,
as he seems easily derailed from his train of thought. A phone call
to a friend listed in the chart provides the additional information
that the man is homeless, and unable to care for himself. This patient
is exhibiting the signs and symptoms of
A. schizoaffective disorder
B. schizoid personality disorder
C. schizophrenia
D. schizophreniform disorder
E. schizotypal personality disorder
Answer
--------------------------------------------------------------------------------

21>Molecular genetic studies are performed on a family with known familial


hypercholesterolemia. In this particular family, the defect in the LDL
receptor gene involves a messenger mutation near the 11th exon, in the
region of homology with epidermal growth factor receptor precursor. A
defect at this site would be most likely to produce which of the following
effects?
CELL BIOLOGY

A. Decreased transcription of LDL receptor gene


B. Poor internalization of LDL bound to LDL receptor
C. Poor retention of the LDL receptor in the membrane
D. Reduced binding of LDL
E. Trapping of the LDL receptor in the endoplasmic reticulum
Answer
--------------------------------------------------------------------------------

22>A 27-year-old woman is giving birth. During the birth, the placental
membranes tear and amniotic fluid is expressed into a lacerated cervical
vein. Which of the following is the woman most likely to experience
immediately following this event?
A. Hemiplegia
B. Placental abruption
C. Renal failure
D. Respiratory distress
E. Splinter hemorrhages
Answer
--------------------------------------------------------------------------------

23>A 30-year-old pregnant woman complains to her physician of feeling very


tired during her pregnancy. A complete blood count with differential
reveals a hematocrit of 30%, with hypersegmented neutrophils and large,
hypochromic red cells. Deficiency of which of the following would be
most likely to produce these findings?
A. Ascorbic acid
B. Calcium
C. Copper
D. Folate
E. Iron
Answer
--------------------------------------------------------------------------------

24>An otherwise healthy 3-year-old child is brought to the pediatrician with


umbilicated, flesh-colored papules on his trunk. This condition is related
to infection with which of the following viruses?
A. Cytomegalovirus
B. Herpesvirus 6
C. Parvovirus
D. Poxvirus
E. Variola
Answer
--------------------------------------------------------------------------------

25>Which of the following renal structures is most medially located?


A. Major calyx
B. Minor calyx
C. Renal cortex
D. Renal pelvis
E. Renal pyramid
Answer
--------------------------------------------------------------------------------

26>A 48-year-old female is being treated for breast carcinoma. Over the past
CELL BIOLOGY

few days, she has been complaining of dysuria and frequency. Laboratory
examination revealed the presence of microscopic hematuria. The next day
the patient developed gross hematuria. Which of the following agents most
likely caused the development of these signs and symptoms?
A. Cyclophosphamide
B. Mitomycin
C. Paclitaxel
D. Tamoxifen
E. Vincristine
Answer
--------------------------------------------------------------------------------

27>A child who has had abnormal development of the membranous bones has a
broad skull with associated facial and dental anomalies. Which other bones
are most likely to also be affected?
A. Clavicles
B. Femurs
C. Metatarsals
D. Phalanges
E. Tibias
Answer
--------------------------------------------------------------------------------

28>A 3-year-old child is referred to a major medical center because of an


abdominal mass arising from his right adrenal gland. Biopsy of the lesion
demonstrates sheets of small cells with hyperchromatic nuclei containing
occasional pseudorosettes composed of circles of tumor cells with central
young nerve fibers arising from the tumor cells. Which of the following
oncogenes is associated with this patient's tumor?
A. erb-B2
B. c-myc
C. L-myc
D. N-myc
E. ret
Answer
--------------------------------------------------------------------------------

29>A 3-year-old child develops headaches and is brought to the family doctor.
Funduscopic examination reveals papilledema; one retina also shows a very
vascular tumor. CT of the head demonstrates a cystic tumor of the
cerebellum. This child has a high likelihood of later developing
which of the following?
A. Berry aneurysm of the basilar system
B. Bilateral renal cell carcinoma
C. Cancer of a peripheral nerve
D. Choreiform movements related to decreased GABA and acetylcholine
E. Serum cholesterol of greater than 700 mg/dL
Answer
--------------------------------------------------------------------------------

30>Administration of an experimental drug that acts on PNS myelin is shown


to increase the space constant of an axon in a peripheral nerve. Action
potentials traveling down the axon would be predicted to be
A. faster
CELL BIOLOGY

B. larger
C. slower
D. smaller
E. unchanged
Answer
--------------------------------------------------------------------------------

31>A 26-year-old man presents to his physician with a chronic cough. The man
is a smoker, and states that he also gets frequent headaches and aches in
his legs when he exercises. Chest x-ray demonstrates notching of his
ribs. Which of the following undiagnosed congenital defects may be
responsible for these findings?
A. Coarctation of the aorta
B. Eisenmenger's syndrome
C. Tetralogy of Fallot
D. Transposition of great vessels
E. Ventricular septal defect
Answer
--------------------------------------------------------------------------------

32>A 36-year-old Asian male complains of difficulty swallowing.


Esophagoscopy reveals a polypoid mass that is subsequently biopsied.
In addition to tumor cells, the esophageal biopsy show normal smooth
muscle and striated muscle in the same section. Which portion of the
esophagus was the source of this biopsy?
A. Lower esophageal sphincter
B. Lower third of the esophagus
C. Middle third of the esophagus
D. Upper esophageal sphincter
E. Upper third of the esophagus
Answer
--------------------------------------------------------------------------------

33> A 25-year-old woman with sickle cell anemia complains of steady pain in
her right upper quadrant with radiation to the right shoulder, especially
after large or fatty meals. Her physician diagnoses gallstones. Of which
of the following compounds are these stones most likely composed?
A. Calcium bilirubinate
B. Calcium oxalate
C. Cholesterol
D. Cholesterol and calcium bilirubinate
E. Cystine
Answer
--------------------------------------------------------------------------------

34>A 54-year-old alcoholic presents with complaints of tremors and muscle


twitching. Physical examination reveals the presence of Trousseau's sign.
Laboratory data show that serum magnesium is < 1 mEq/L
(normal, 1.4 - 2.2 mEq/L). Which of the following findings
would be most consistent with this information?
A. Decreased serum calcium
B. Decreased serum phosphate
CELL BIOLOGY

C. Increased bone density


D. Increased plasma parathyroid hormone concentration
E. Increased urinary cAMP concentration
Answer
--------------------------------------------------------------------------------

35>A young boy presents with failure to thrive. Biochemical analysis of a


duodenal aspirate after a meal reveals a deficiency of enteropeptidase
(enterokinase). The levels of which of the following digestive enzymes
would be affected?
A. Amylase
B. Colipase
C. Lactase
D. Pepsin
E. Trypsin
Answer
--------------------------------------------------------------------------------

36>A Guatemalan child with a history of meconium ileus is brought to a


clinic because of a chronic cough. The mother notes a history of respiratory
tract infections and bulky, foul-smelling stools. After assessment of the
respiratory tract illness, the physician should also look for signs of
A. cystinuria
B. hypoglycemia
C. iron deficiency anemia
D. sphingomyelin accumulation
E. vitamin A deficiency
Answer
--------------------------------------------------------------------------------
Answers
--------------------------------------------------------------------------------
1>The correct answer is A. A number of physiologic changes occur in a person living at high altitude. The diminished
barometric pressure at high altitude causes alveolar hypoxia and arterial hypoxia. Pulmonary vasoconstriction occurs
in response to alveolar hypoxia; therefore, the diameter of the pulmonary vessels would be greater in the brother
living at sea level. All the other choices describe physiologic processes that would be enhanced by living at high
altitude.
Increased erythropoietin production (choice B), caused by arterial hypoxia, leads to increases in hematocrit in people
living at high altitude.
Mitochondrial density increases (choice C) in people chronically exposed to the hypoxemia caused by living at high
altitude.
At high altitudes, the ventilation rate increases, causing a respiratory alkalosis. The kidney then compensates by
increasing the excretion of HCO3- (choice D).
Increasing the rate of respiration (choice E) is a very useful adaptation to the hypoxic conditions of high altitude. The
primary stimulus is the hypoxic stimulation of peripheral chemoreceptors.
--------------------------------------------------------------------------------
2>The correct answer is A. Alopecia areata is caused by an autoimmune attack on hair follicles. It has a wide range
of clinical severity, with most cases involving a localized patch of hair (which regrows within 1 year in half of the
patients). The hair that does regrow may be gray or depigmented. More severe cases can involve the entire scalp
(alopecia totalis) or, as in this patient, the entire body surface (alopecia universalis). These more severe cases are
less likely to resolve adequately. Treatment of alopecia areata is often unsuccessful, but topical steroids are typically
tried.
Androgenic alopecia (choice B) is common male pattern baldness.
Chronic cutaneous lupus erythematosus (choice C) can produce localized baldness.
CELL BIOLOGY

Lichen planopilaris (choice D) can produce localized baldness.


Trichotillomania (choice E), also called traumatic alopecia, is alopecia due to trauma, such as hair pulling or tight
braids.
--------------------------------------------------------------------------------3>The correct answer is B. Both the omental bursa
and the greater omentum are derived from the dorsal mesogastrium, which is the mesentery of the stomach region.
The dorsal mesoduodenum (choice A) is the mesentery of the developing duodenum, which later disappears so that
the duodenum and pancreas come to lie retroperitoneally.
The pericardioperitoneal canal (choice C) embryologically connects the thoracic and peritoneal canals.
The pleuropericardial membranes (choice D) become the pericardium and contribute to the diaphragm.
The ventral mesentery (choice E) forms the falciform ligament, ligamentum teres, and lesser omentum.
--------------------------------------------------------------------------------4>The correct answer is C. Wine and cheese (and
many other fermented foods) contain tyramine, an indirect sympathomimetic that can trigger excess catecholamine
release and lead to a hypertensive crisis when ingested by patients taking MAO inhibitors. The only such drug listed
among the answer choices is phenelzine. Other MAO inhibitors with similar effects include tranylcypromine,
isocarboxazid, and iproniazid. Whenever this particular drug class is mentioned in a question stem, consider the
possibility of interactions with foods or other medications the patient may have taken.
Amitriptyline (choice A) is a tricyclic antidepressant. Tricyclic antidepressant drugs (particularly amitriptyline) are
known for their anticholinergic side effects. They also produce postural hypotension (because they block alpha-
adrenergic receptors) and are sedative.
Fluoxetine and sertraline (choices B and D) are antidepressants that are selective serotonin reuptake inhibitors
(SSRIs). Fluoxetine is also useful in treatment of obsessive-compulsive disorders.
Trazodone (choice E) is an atypical antidepressant with substantial sedative side effects. Its most serious side effect
is priapism, a medical emergency.
--------------------------------------------------------------------------------5>The correct answer is D. The cells of the anterior
pituitary can be classified as chromophils (love dyes) or chromophobes (do not stain with dyes). The chromophils can
be further divided into acidophils (stain with acidic dyes) and basophils (stain with basic dyes). The acidophils include
the somatotropes, which secrete growth hormone, and the mammotropes, which secrete prolactin. The basophils
include the corticotropes, which secrete ACTH (choice A), the gonadotropes, which secrete FSH and LH (choices B
and C), and the thyrotropes, which secrete TSH (choice E).
--------------------------------------------------------------------------------6>The correct answer is D. Persons with this
condition often perceive attacks and danger in relatively innocuous situations. They are quick to respond with anger,
and, because personality disorders are ego-syntonic, individuals with personality disorders do not believe themselves
to be in error.
The individual with dependent personality disorder (choice A) does not confront others but wants others to take care
of him.
The individual with histrionic personality disorder (choice B) is flamboyant and seductive, not confrontational and
angry.
The individual with narcissistic personality disorder (choice C) is characterized by feelings of entitlement because they
are so "special."
And the individual with passive aggressive personality disorder (choice E) expresses anger indirectly (e.g., always
being late) rather than confronting
--------------------------------------------------------------------------------7>The correct answer is C. The loading of O2 is
facilitated when the oxygen dissociation curve shifts to the left, and the unloading of O2 is facilitated when the
oxygen dissociation curve shifts to the right. A good way to remember the conditions that promote dissociation of O2
is to think of exercising muscle, which has decreased pH (choice C) because of the accumulation of lactic acid,
increased PCO2 (compare with choice B) because of the increased rate of aerobic metabolism, increased temperature
(compare with choice D), and increased 2,3-DPG (2,3-diphosphoglycerate; compare with choice E) because of
increased glycolysis.
Carbon monoxide poisoning (choice A) left-shifts the oxygen dissociation curve, which interferes with the unloading
of O2. Carbon monoxide also strongly binds to available sites on hemoglobin.
--------------------------------------------------------------------------------8>The correct answer is A. This patient has
Huntington's disease, which has autosomal dominant inheritance. It is characterized by severe degeneration of the
CELL BIOLOGY

caudate nucleus along with degenerative changes in the putamen and cortex. In addition to chorea, these patients
frequently suffer from athetoid (writhing) movements, progressive dementia, and behavioral disorders.
--------------------------------------------------------------------------------9>The correct answer is E. The rash described is
that of secondary syphilis, caused by Treponema pallidum. Involvement of palms and soles by a rash is unusual, and
secondary syphilis should come to mind. Not all patients with secondary syphilis have a severe form of the rash, and
consequentially some cases are missed. Primary syphilis takes the form of a painless, button-like mass called
chancres. Tertiary syphilis, which is now rare, has a propensity for involving the aorta and central nervous system
and can also cause "gummas" (granulomatous-like lesions) in many sites, notably including liver and bone.
Herpes simplex I (choice A) usually causes perioral vesicular lesions.
Herpes simplex II (choice B) usually causes genital vesicular lesions.
HIV (choice C) does not itself cause a rash, although co-infection with other organisms can result in a rash.
Neisseria gonorrhoeae (choice D) does not typically cause a rash.
--------------------------------------------------------------------------------10>The correct answer is B. Hyperlipidemia has
been subclassified based on the lipid and lipoprotein profiles. Type 2a, which this patient has, can be seen in a
hereditary form, known as familial hypercholesterolemia, and also in secondary, acquired forms related to nephritic
syndrome and hyperthyroidism. The root problem appears to be a deficiency of LDL receptors, which leads to a
specific elevation of cholesterol in the form of increased LDL. Heterozygotes for the hereditary form generally
develop cardiovascular disease from 30 to 50 years of age. Homozygotes may have cardiovascular disease in
childhood.
Type 1 (choice A) is characterized by isolated elevation of chylomicrons.
Type 2b (choice C) is characterized by elevations of both cholesterol and triglycerides in the form of LDL and VLDL.
Type 3 (choice D) is characterized by elevations of triglycerides and cholesterol in the form of chylomicron remnants
and IDL.
Type 5 (choice E) is characterized by elevations of triglycerides and cholesterol in the form of VLDL and
chylomicrons.
--------------------------------------------------------------------------------11The correct answer is C. The neurohypophysis
(posterior pituitary) is derived from an evagination of diencephalic neurectoderm. This structure is responsible for
releasing oxytocin and vasopressin to the general circulation. Both hormones are synthesized in cell bodies contained
within the hypothalamus.
ACTH (choice A), prolactin (choice D), and TSH (choice E) are all synthesized and released by the anterior pituitary,
or adenohypophysis, which is derived from an evagination of the ectoderm of Rathke's pouch, a diverticulum of the
primitive mouth. Remnants of this pouch may give rise to a craniopharyngioma in later life.
Epinephrine (choice B) is synthesized and released into the circulation by the adrenal medulla, a neural crest
derivative.
--------------------------------------------------------------------------------12>The correct answer is E. The most common
bacteria implicated in community-acquired pneumonia is the pneumococcus, Streptococcus pneumoniae. Other
organisms frequently implicated in patients less than age 60 without comorbidity include Mycoplasma pneumoniae,
respiratory viruses, Chlamydia pneumoniae, and Haemophilus influenzae. When community-acquired pneumonia
occurs in elderly patients or patients with comorbidity, aerobic gram-negative bacilli and Staphylococcus aureus are
added to the list.
The organisms listed in choices A, B, and C are important causes of community-acquired pneumonia, but are not the
most frequent causes.
Staphylococcus aureus (choice D) is an important cause of community-acquired pneumonia (particularly in the elderly
and in patients with comorbidity), but is not the most frequent cause.
--------------------------------------------------------------------------------13>The correct answer is E. Uric acid kidney stones
in patients with leukemia are secondary to increased production of uric acid from purine breakdown during periods of
active cell proliferation, especially following treatment. Vigorous hydration and diuresis are generally instituted after
the diagnosis of acute leukemia is made. Uric acid kidney stones are also associated with inborn errors of purine
metabolism, such as gout.
Pigment gallstones (choice A) are associated with hemolytic disease. The incidence of this type of gallstone is not
increased in treated leukemias.
Cholesterol gallstones (choice B) are associated with diabetes mellitus, obesity, pregnancy, birth control pills, and
CELL BIOLOGY

celiac disease.
Cystine kidney stones (choice C) are rare; they are found in cystinuria.
Struvite kidney stones (choice D) are associated with infection by urea-splitting organisms, such as Proteus.
--------------------------------------------------------------------------------14>The correct answer is B. The data shown in the
table indicate that the man has developed metabolic alkalosis (increased PCO2, pH, and HCO3-), which occurs
commonly with overuse of diuretics (thiazides and loop diuretics). The overuse of a loop diuretic increases the
excretion of sodium (choice E) and potassium (choice C) by the kidneys. The increase in potassium excretion leads to
a decrease in plasma potassium levels (choice B). The decrease in plasma potassium stimulates aldosterone
secretion, which raises plasma aldosterone levels (choice A). The sodium depletion stimulates renin secretion (choice
D), which in turn raises angiotensin II levels in the plasma (which also stimulates aldosterone secretion
--------------------------------------------------------------------------------15>The correct answer is C. This is a case of
Reiter's syndrome. Patients typically present with the acute onset of arthritis (usually asymmetric and additive), with
involvement of new joints occurring over a period of a few days to 2 weeks. Joints of the lower extremities are the
most commonly involved, but wrists and fingers can also be affected. Dactylitis (sausage digit), a diffuse swelling of a
solitary finger or toe, is a distinctive feature of Reiter's arthritis and psoriatic arthritis. Tendonitis and fasciitis are
common. Spinal pain and low back pain are common. Conjunctivitis, urethritis, diarrhea, and skin lesions are also
associated with Reiter's syndrome. Up to 75% of patients are HLA-B27 positive. Microorganisms which can trigger
Reiter's syndrome include Shigella spp., Salmonella spp., Yersinia spp., Campylobacter jejuni, and Chlamydia
trachomatis. Most patients are younger males.
Gout (choice A) usually presents as an explosive attack of acute, very painful, monarticular inflammatory arthritis.
Hyperuricemia is the cardinal feature and prerequisite for gout. The first metatarsophalangeal joint is involved in over
50% of first attacks.
Lyme disease (choice B), caused by Borrelia burgdorferi, presents with a red macule or papule at the site of the tick
bite. This lesion, called erythema chronicum migrans, slowly expands to form a large annular lesion with a red border
and central clearing. The lesion is warm, but usually not painful. The patient also has severe headache, stiff neck,
chills, arthralgias, and profound malaise and fatigue. Untreated infection is associated with development of arthritis.
The large joints (e.g., knees) are usually involved with the arthritis lasting for weeks to months.
Rheumatoid arthritis (choice D) begins insidiously with fatigue, anorexia, generalized weakness, and vague
musculoskeletal symptoms leading up to the appearance of synovitis. Pain in the affected joints, aggravated by
movement, is the most common manifestation of established rheumatoid arthritis. Generalized stiffness is frequent
and is usually greatest after periods of inactivity. Morning stiffness of greater than 1 hour in duration is very
characteristic. Rheumatoid arthritis is more common in females. The metacarpophalangeal and proximal
interphalangeal joints of the hands are characteristically involved.
Septic arthritis (choice E) is caused by a variety of microorganisms, including Neisseria gonorrhoeae and
Staphylococcus aureus. Hematogenous spread is the most common route in all age groups. 90% of patients present
with involvement of a single joint, usually the knee. The usual presentation is moderate-to-severe pain, effusion,
muscle spasm, and decreased range of motion. Peripheral leukocytosis and a left shift are common. Disseminated
gonococcal infections present as fever, chills, rash, and articular symptoms. Papules progressing to hemorrhagic
pustules develop on the trunk and extensor surfaces of the distal extremities. Migratory arthritis and tenosynovitis of
multiple joints is common.
--------------------------------------------------------------------------------16>The correct answer is D. Many people who
request physician-assisted suicide have one of two conditions present: either a poorly controlled painful condition or
severe depression. If the painful condition is adequately treated or the depression is brought under good medical
control, the request for physician assistance in terminating the situation is typically withdrawn. It is important to note
that bringing these conditions under control requires the intervention of caregivers who are specifically trained in the
management of these two conditions; primary care physicians usually are not adequately trained to address these
difficult presentations.
While patients who are diagnosed as bipolar disorder (choice A), borderline personality disorder (choice B), and
schizophrenic disorder (choice E) often make suicide attempts (and frequently complete those attempts), they do not
generally ask their physician for assistance in the suicide.
Persons with factitious disorder (choice C) are seeking primary gain, often for dependency needs, and are seeking to
enter the "sick role" not the "dead role."
CELL BIOLOGY

--------------------------------------------------------------------------------17>The correct answer is E. The patient has


bilateral acoustic neuromas, probably due to neurofibromatosis type II (over 90% of patients with NF-2 develop
bilateral acoustic neuromas). This condition is a associated with the NF-2, gene, located on 22q (note all the 2's).
Patients often develop meningiomas, gliomas, and schwannomas of cranial and spinal nerves.
5q (choice A) contains the APC tumor suppressor gene, which is associated with familial and sporadic colorectal
cancers.
13q (choice B) contains the Rb tumor suppressor gene, which is associated with retinoblastoma and osteosarcoma.
17q (choice C) contains both the NF-1 tumor suppressor gene, which is associated with neurofibromatosis type I, and
the p53 tumor suppressor gene, associated with many human cancers.
18q (choice D) contains both the DCC gene, which is associated with colon and gastric carcinomas and the DPC
gene, associated with pancreatic cancer.

--------------------------------------------------------------------------------18>
The correct answer is D. Perforation of a peptic ulcer is potentially fatal, because of either peritonitis with sepsis or
sudden exsanguination (if the perforation damages one of the many arteries of the stomach). Peptic ulcer disease,
gastritis, and possibly gastric carcinoma and gastric lymphoma have been strongly associated with Helicobacter pylori
colonization of the mucus layer covering the gastric mucosa. Colonization is associated with destruction of the mucus
layer, thereby destroying its protective function.
Cryptosporidium parvum (choice A) causes diarrhea that is severe in immunocompromised patients.
Entamoeba histolytica (choice B) produces dysentery-like symptoms or can cause liver abscess.
Escherichia coli (choice C) causes a variety of diarrheal diseases and can infect the bladder and soft tissues.
Mycobacterium tuberculosis (choice E) causes tuberculosis, characterized by granuloma formation, especially in the
lungs.
--------------------------------------------------------------------------------19>The correct answer is E. By the third week of
development, hematopoiesis begins in the blood islands of the yolk sac. Beginning at 1 month of age and continuing
until 7 months of age, blood elements are also formed in the liver. Hematopoiesis occurs in the spleen and lymphatic
organs between 2 and 4 months, and in the bone marrow after 4 months.
--------------------------------------------------------------------------------
15>The correct answer is C. The patient is suffering from schizophrenia. The key to the diagnosis of psychosis is that
there has been a marked decline in the level of functioning (i.e., the man is homeless and cannot care for himself).
Although hallucinations or delusions are not mentioned in the case history, the presence of disorganized speech,
grossly disorganized behavior, and the duration of symptoms (longer than six months) suggest a diagnosis of
schizophrenia.
In schizoaffective disorder (choice A), alterations in mood are present during a substantial portion of the illness.
Although schizoid personality disorder (choice B) produces detachment from social relationships and is characterized
by restriction of emotional expression, it is not accompanied by a marked decline in occupational functioning.
Schizophreniform disorder (choice D) is characterized by schizophrenic-like symptoms, but the duration of symptoms
is, by definition, less than six months.
Schizotypal personality disorder (choice E) is characterized by eccentricities of behavior, odd beliefs or magical
thinking, and difficulties with social and interpersonal relationships. Unlike schizophrenia, schizotypal personality
disorder is not characterized by a formal thought disorder.
--------------------------------------------------------------------------------

--------------------------------------------------------------------------------21>The correct answer is E. Familial


hypercholesterolemia, which is due to defective function of the LDL receptor, is an area of intense research. The
molecular basis of LDL receptor abnormalities is becoming better understood, and more than 200 mutations in the
gene for the LDL receptor have been identified. The gene has 5 general domains and 18 exons. Defects near exons 7
to 14 (including this case) are in the region of homology with epidermal growth factor receptor precursor. This region
of the molecule is needed for dissociation of LDL from the receptor in the endosome. Receptors with a defect in this
area (sometimes called class II mutations) also have trouble being initially transported to the Golgi complex
(transport-deficiency alleles) and become trapped in endoplasmic reticulum.
CELL BIOLOGY

Decreased transcription of the LDL receptor gene (choice A) is considered a class I mutation and involves the signal
sequence domain near exon 1.
Poor internalization of LDL bound to LDL receptor (choice B) is considered a class IV mutation. Such mutations are
associated with the membrane-spanning/cytoplasmic domain, specifically near exon 18.
Poor retention of the LDL receptor in the membrane (choice C) is considered a class IV mutation and is associated
with the membrane-spanning/cytoplasmic domain, specifically near exons 2-6.
Reduced binding of LDL (choice D) is considered a class III mutation and involves the LDL binding domain near
exons 2-6.
--------------------------------------------------------------------------------22>The correct answer is D. Respiratory distress
immediately follows amniotic fluid embolism as the emboli consisting of squamous cells, lanugo, and mucus deposit
in the pulmonary microcirculation, producing numerous tiny pulmonary infarcts. The dramatic respiratory distress
may also reflect the action of prostaglandins and other bioactive compounds present in high concentrations in the
amniotic fluid embolus.
Hemiplegia (choice A) would reflect an ischemic injury to one hemisphere of the cerebrum or the brainstem. A
venous embolus would not produce such an insult.
Placental abruption (choice B) is partial, premature separation of the placental disc from the endometrium. Although
abruption may occur in this setting, it is not a result of an amniotic fluid embolism.
There are numerous causes of renal failure (choice C); the most likely ones in the peripartum interval include
eclampsia, hypovolemic shock, and ascending infections. Amniotic fluid embolism would be expected to produce
severe dyspnea well before shock and renal failure might arise.
Splinter hemorrhages (choice E) are small hemorrhages seen on toes and fingers due to a shower of microemboli
arising in the arterial circulation. Amniotic fluid emboli arise in the veins and deposit in the lungs.
--------------------------------------------------------------------------------23>The correct answer is D. The patient has a
megaloblastic anemia, which can be due to deficiency of folate or B12. Pregnancy increases the need for folate and
other nutrients used by both baby and mother, and may "unmask" a borderline dietary deficiency. For this reason,
most obstetricians recommend vitamin supplements for pregnant women.
Ascorbic acid (choice A) is vitamin C, and its deficiency predisposes for capillary fragility and oral lesions.
Calcium deficiency (choice B) predisposes for osteoporosis/osteopenia.
Copper deficiency (choice C) is rare; when it occurs, it may cause a hypochromic anemia, neutropenia, osteoporosis,
or hypotonia.
Iron deficiency (choice E) causes a microcytic, hypochromic anemia, with reduced mental and physical performance.
--------------------------------------------------------------------------------24>The correct answer is D. The lesions are
characteristic of molluscum contagiosum, which is a typically benign and self-limited condition caused by a poxvirus.
The disease can be transmitted either venereally or through non-venereal contact. The other viruses listed do not
cause similar skin lesions. Patients with advanced HIV infection may develop a severe, generalized, and persistent
eruption, often involving the face and upper body.
Cytomegalovirus (choice A) causes congenital infections and disseminated infections in immunosuppressed patients.
Herpesvirus 6 (choice B)causes roseola (exanthem subitum).
Parvovirus (choice C) causes aplastic crises in patients with hemolytic anemia.
Variola (choice E) is the smallpox virus.
--------------------------------------------------------------------------------25>The correct answer is D. This is a relatively
simple question that requires you to visualize the relationship among the key parts of the kidney and to identify the
one that lies most medially. If you think about it for a second, since the kidneys ultimately drain into the ureter at
their medial poles, you are looking for the structure that is closest to the ureter. The correct answer is the renal
pelvis. The renal pelvis is the dilated upper portion of the ureter that receives the major calyces.
In terms of the other answer choices, the order from most lateral to most medial is: renal cortex (choice C), renal
pyramid (choice E), minor calyx (choice B), major calyx (choice A), and then the renal pelvis (choice D).
--------------------------------------------------------------------------------
26>
The correct answer is A. Cyclophosphamide is metabolized to acrolein, which is excreted in the urine. If the patient's
urine is concentrated, the toxic metabolite may cause severe bladder damage. Early symptoms of bladder toxicity
include dysuria and frequency. This can be distinguished from a urinary tract infection, since there is no bacteriuria
CELL BIOLOGY

with cyclophosphamide-induced bladder toxicity. However, microscopic hematuria is often present on urinalysis. In
severe hemorrhagic cystitis, large segments of the bladder mucosa may be shed which can lead to prolonged, gross
hematuria. The incidence of cyclophosphamide-induced hemorrhagic cystitis can be decreased by ensuring that the
patient maintains a high fluid intake. Cyclophosphamide is an alkylating agent used in the treatment of breast
carcinoma, malignant lymphoma, multiple myeloma, and adenocarcinoma of the ovary, as well as various other
forms of cancer. The major toxic reactions commonly seen with this agent include mucositis, nausea, hepatotoxicity,
sterile hemorrhagic and non-hemorrhagic cystitis, leukopenia, neutropenia, and interstitial pulmonary fibrosis.
Mitomycin (choice B) is an antibiotic antineoplastic agent used in the treatment of breast carcinoma, adenocarcinoma
of the pancreas and stomach, as well as various other forms of cancer. The major toxic reactions commonly seen
with this agent include bone marrow depression, nausea, hepatotoxicity, acute bronchospasm, thrombocytopenia,
and interstitial pneumonitis.
Paclitaxel (choice C) is an antineoplastic agent primarily used in the treatment of ovarian and breast cancer. The
major toxic reactions commonly seen with this agent include bone marrow depression, nausea, hepatotoxicity,
bronchospasm, thrombocytopenia, and neutropenia.
Tamoxifen (choice D) is an antineoplastic hormone primarily used in the palliative treatment of estrogen-receptor
positive breast cancer patients. The major toxic reactions commonly seen with this agent include depression,
dizziness, thrombosis, mild leukopenia or thrombocytopenia.
Vincristine (choice E) is a mitotic inhibitor antineoplastic agent used in the treatment of breast cancer, Hodgkin's
disease, non-Hodgkin's lymphoma, advanced testicular cancer and various other types of cancer. The major toxic
reactions commonly seen with this agent include mental depression, hemorrhagic enterocolitis, bone marrow
depression, nausea, thrombocytopenia, and leukopenia
--------------------------------------------------------------------------------27>The correct answer is A. In a syndrome called
cleidocranial dysostosis, absence of part of the clavicles accompanies a broad skull, and facial and dental anomalies.
Note that you could also have answered this question by noting that of the bones listed, only the clavicles form by
intramembranous ossification.
The femurs (choice B), metatarsals (choice C), phalanges (choice D), and tibias (choice E) are cartilaginous (formed
by endochondral ossification) rather than membranous bones.
--------------------------------------------------------------------------------28>The correct answer is D. The tumor is a
neuroblastoma, which is one of the principal forms of cancer in children. Neuroblastoma typically occurs before age
5, with many presenting before age 2. Neuroblastoma can arise from neural crest cells throughout the body, but the
adrenal medulla is the most common site. Homer-Wright pseudorosettes are circles of tumor cells with central young
nerve fibers arising from the tumor cells. The oncogene associated with neuroblastoma is N-myc.
erb-B2 (choice A) is associated with breast, ovarian, and gastric carcinomas.
c-myc (choice B) is associated with Burkitt's lymphoma.
L-myc (choice C) is associated with small cell carcinoma of the lung.
ret (choice E) is associated with multiple endocrine neoplasia, types II and III.
--------------------------------------------------------------------------------29>The correct answer is B. The disease is von
Hippel-Landau disease, which is associated with a deletion involving the VHL gene on chromosome 3 (3p). Affected
individuals develop vascular tumors (hemangioblastomas) of the retina, cerebellum, and/or medulla. Roughly half of
the affected individuals later develop multiple, bilateral renal cell carcinomas.
Berry aneurysms (choice A) are unrelated to hemangioblastomas, but are instead associated with adult polycystic
disease.
Peripheral nerve cancers (choice C) are a feature of von Recklinghausen's disease (neurofibromatosis type I).
Choreiform movements, related to decreased GABA and acetylcholine (choice D), are a feature of Huntington's
disease.
Extremely high serum cholesterol (choice E) suggests the homozygous form of familial hypercholesterolemia.
--------------------------------------------------------------------------------30>The correct answer is A. The space constant of
an axon reflects the amount of passive or electrotonic spread of current within an axon. The larger the space
constant, the further the current can spread, allowing action potentials to propagate faster. This is why myelin
increases the conduction velocity of action potentials down an axon. Conversely, demyelination decreases the space
constant and slows action potential conduction.
--------------------------------------------------------------------------------31>The correct answer is A. Coarctation of the
CELL BIOLOGY

aorta occurs in two patterns. In the infantile type, the stenosis is proximal to the insertion of the ductus arteriosus
(preductal); this pattern is associated with Turner's syndrome. In the adult form, the stenosis is distal to the ductus
arteriosus (postductal) and is associated with notching of the ribs (secondary to continued pressure from the aorta
on them), hypertension in the upper extremities, and weak pulses in the lower extremities. Headache, cold
extremities, and lower extremity claudication with exercise are typical if the patient is symptomatic (many adults with
mild distal coarctation may remain asymptomatic for years). Upper extremity hypertension with weak pulses in the
lower extremities, and a midsystolic (or continuous) murmur over the chest or back may be the only obvious signs in
some. Note that the chronic cough is probably related to the man's smoking, and is not caused by the coarctation.
Eisenmenger's syndrome (choice B) is a shift from a left-to-right shunt to a right-to-left shunt secondary to
pulmonary hypertension.
Tetralogy of Fallot (choice C) and transposition of great arteries (choice D) cause cyanosis and are usually diagnosed
in infancy.
Ventricular septal defect (choice E) might remain undiagnosed until adulthood, but would not cause notching of the
ribs.
--------------------------------------------------------------------------------32>The correct answer is C. The muscularis of the
upper third of the esophagus (choice E) is composed entirely of striated muscle. The middle third (choice C) contains
both striated and smooth muscle. The lower third (choice B) and lower esophageal sphincter (choice A) contain only
smooth muscle. There is no such thing as the upper esophageal sphincter (choice D).
--------------------------------------------------------------------------------33>The correct answer is A. Bilirubin is a
degradative product of hemoglobin metabolism. Bilirubin (pigment) stones are specifically associated with excessive
bilirubin production in hemolytic anemias, including sickle cell anemia. Bilirubin stones can also be seen in hepatic
cirrhosis and liver fluke infestation.
Calcium oxalate stones (choice B) and cystine stones (choice E) are found in the kidney, rather than the gallbladder.
Pure cholesterol stones (choice C) are less common than mixed gallstones, but have the same risk factors, including
obesity and multiple pregnancies.
Mixed stones (choice D) are the common "garden variety" gallstones, found especially in obese, middle aged
patients, with a female predominance
--------------------------------------------------------------------------------
34>
The correct answer is A. Malnutrition associated with chronic alcoholism can lead to a severe magnesium deficiency.
The effect of low serum magnesium on parathyroid hormone secretion (PTH) depends on severity and duration. An
acute decrease in serum magnesium will increase PTH secretion, while a prolonged severe deficiency results in
decreased PTH secretion. There is also evidence that the action of PTH is decreased with chronic magnesium
deficiency. Hence, this patient is suffering from "functional" hypoparathyroidism. The low serum calcium can produce
weakness, tremors, muscle fasciculations, and seizures. A positive Trousseau's sign indicates the presence of latent
tetany. It is observed by inflating a blood pressure cuff above systolic blood pressure for at least 2 minutes. A
positive reaction consists of the development of carpal spasm, with relaxation occurring within seconds after deflating
the cuff. In patients with magnesium deficiency, magnesium administration will produce a prompt rise in plasma PTH
with subsequent restoration of serum calcium concentration to normal.
With functional hypoparathyroidism bone density would be decreased (not increased, choice C).
The combination of decreased PTH secretion (not increased, choice D) and decreased effectiveness of PTH produce
hypocalcemia and hyperphosphatemia (not hypophosphatemia, choice B).
Urinary cAMP would probably be decreased (not increased, choice E), given the low PTH.
--------------------------------------------------------------------------------35>The correct answer is E. Enteropeptidase,
formerly called enterokinase, activates trypsinogen by limited proteolytic digestion to give trypsin. Trypsin is itself
capable of activating trypsinogen, which produces a positive feedback effect. Trypsin also activates
chymotrypsinogen (and several other proteolytic enzymes), so deficiency of enteropeptidase results in a severe
deficiency of enzymes that digest protein.
Amylase (choice A) aids in the breakdown of starches to oligosaccharides, maltose, and maltotriose.
Colipase (choice B), along with other lipases, functions to digest fats.
Lactase (choice C) is a brush-border disaccharidase that hydrolyzes the bond between galactose and glucose in
lactose.
CELL BIOLOGY

Pepsin (choice D) is a proteolytic enzyme secreted in an inactive form (pepsinogen) by the chief cells of the stomach.
Pepsinogen is activated by stomach acid, and so is not dependent on enteropeptidase. Pepsin alone will not replace
the activities of other proteolytic enzymes, partly because food does not remain in the stomach for an extended
period of time.
--------------------------------------------------------------------------------36>The correct answer is E. The child is likely
suffering from cystic fibrosis. In this disorder, an abnormality of chloride channels causes all exocrine secretions to be
much thicker, and more viscous than normal. Pancreatic secretion of digestive enzymes is often severely impaired,
with consequent steatorrhea and deficiency of fat-soluble vitamins, including vitamin A.
Cystinuria (choice A) is a relatively common disorder in which a defective transporter for dibasic amino acids (cystine,
ornithine, lysine, arginine; COLA) leads to saturation of the urine with cystine, which is not very soluble in urine, and
precipitates out to form stones.
Hypoglycemia (choice B) is not a prominent feature of children with cystic fibrosis who are on a normal diet.
Hyperglycemia may occur late in the course of the disease.
Iron deficiency anemia (choice C) is not found with any regularity in children with cystic fibrosis.
Sphingomyelin accumulation (choice D) is generally associated with deficiency of sphingomyelinase, as seen in
Niemann-Pick disease.
--------------

BIOCHEMISTRY

A 25-year-old woman with sickle cell anemia complains of steady pain in her right upper quadrant with
radiation
to the right shoulder, especially after large or fatty meals. Her physician diagnoses gallstones. Of
which of the
following compounds are these stones most likely composed?

A. Calcium bilirubinate

B. Calcium oxalate

C. Cholesterol

D. Cholesterol and calcium bilirubinate

E. Cystine

Explanation:

The correct answer is A. Bilirubin is a degradative product of hemoglobin metabolism. Bilirubin


(pigment) stones
are specifically associated with excessive bilirubin production in hemolytic anemias, including sickle
cell anemia.
Bilirubin stones can also be seen in hepatic cirrhosis and liver fluke infestation.

Calcium oxalate stones (choice B) and cystine stones (choice E) are found in the kidney, rather than
the
gallbladder.

Pure cholesterol stones (choice C) are less common than mixed gallstones, but have the same risk
CELL BIOLOGY

factors,
including obesity and multiple pregnancies.

Mixed stones (choice D) are the common "garden variety" gallstones, found especially in obese,
middle aged
patients, with a female predominance.

Two sisters are diagnosed with hemolytic anemia. Their older brother was previously diagnosed with
the same
disorder. Two other brothers are asymptomatic. The mother and father are second cousins. Deficiency
of which of
the following enzymes would be most likely to cause this disorder?

A. Debranching enzyme

B. Glucose-6-phosphatase

C. Glucose-6-phosphate dehydrogenase

D. Muscle phosphorylase

E. Pyruvate kinase

Explanation:

The correct answer is E. In general, you should associate hemolytic anemia with defects in glycolysis
or the
hexose monophosphate shunt (pentose phosphate pathway). Only two enzymes of those listed in the
answer
choices specifically involve these pathways and cause hemolytic anemia: pyruvate kinase and
glucose-6-phosphate dehydrogenase. Glucose-6-phosphate dehydrogenase (G6PD) deficiency is
inherited as
an X-linked recessive trait, so females would not be affected. Pyruvate kinase is a glycolytic enzyme;
pyruvate
kinase deficiency is an autosomal recessive disorder, affecting males and females approximately
equally. If this
enzyme is deficient, red cells have trouble producing enough ATP to maintain the Na+/K+ pump on
the plasma
membrane, secondarily causing swelling and lysis.

Debranching enzyme (choice A) defects produce Cori's disease, one of the glycogen storage diseases.

Defects in glucose-6-phosphatase (choice B) produce Von Gierke's disease, one of the glycogen
storage
diseases.

Glucose-6-phosphatase dehydrogenase (choice C) deficiency produces an X-linked hemolytic anemia.

Defects in muscle phosphorylase (choice D) produce McArdle's disease, one of the glycogen storage
diseases.
CELL BIOLOGY

EMBRYOLOGY

The greater omentum is derived from which of the following embryonic structures?

A. Dorsal mesoduodenum

B. Dorsal mesogastrium

C. Pericardioperitoneal canal

D. Pleuropericardial membranes

E. Ventral mesentery

Explanation:

The correct answer is B. Both the omental bursa and the greater omentum are derived from the dorsal
mesogastrium, which is the mesentery of the stomach region.

The dorsal mesoduodenum (choice A) is the mesentery of the developing duodenum, which later
disappears so that the duodenum and pancreas come to lie retroperitoneally.

The pericardioperitoneal canal (choice C) embryologically connects the thoracic and peritoneal canals.

The pleuropericardial membranes (choice D) become the pericardium and contribute to the diaphragm.

The ventral mesentery (choice E) forms the falciform ligament, ligamentum teres, and lesser omentum.

A PSA level is drawn from a 54-year-old man as part of a routine health evaluation. Which of the
following embryonic structures gives rise to the organ being screened for carcinoma?

A. Genital tubercle

B. Processus vaginalis

C. Testis cords

D. Tunica albuginea

E. Urogenital sinus

Explanation:

The correct answer is E. It is recommended that PSA levels (prostate-specific antigen) be


measured annually in men over age 50 to screen for prostatic carcinoma (and to record a
baseline level). The prostate is immediately derived from the prostatic urethra, which is
derived from the urogenital sinus.

The genital tubercle (choice A) gives rise to the glans penis.

The processus vaginalis (choice B) is a coelomic extension into the scrotal swelling that
carries with it extensions of the body wall to form the inguinal canal during the descent of the testes.

The testis cords (choice C) are composed of primitive germ cells, which give rise to
spermatogonia, and sex cord cells, which differentiate into Sertoli cells.

The tunica albuginea (choice D) is derived from mesenchyme and condenses to form the fibrous
connective tissue capsule of the testis.

A newborn baby has projectile vomiting shortly after each feeding. It is determined that there is
obstruction of the digestive tract as a result of an annular pancreas. Annular pancreas is a
result of an abnormality in which of the following processes?
CELL BIOLOGY

A. Rotation of the dorsal pancreatic bud around the first part of the duodenum

B. Rotation of the dorsal pancreatic bud around the second part of the duodenum

C. Rotation of the dorsal pancreatic bud around the third part of the duodenum

D. Rotation of the ventral pancreatic bud around the first part of the duodenum

E. Rotation of the ventral pancreatic bud around the second part of the duodenum

Explanation:

The correct answer is E. The ventral pancreatic bud normally rotates around the duodenum to
fuse with the dorsal pancreatic bud. Both pancreatic buds form from evaginations from the
second part of the duodenum, hence the rotation is around the second part of the duodenum. The
normal rotation is around the right side of the embryonic duodenum. Annular pancreas results
from the ventral pancreatic bud dividing and rotating around both the right and left sides of
the second part of the duodenum, thus encircling it.

The dorsal pancreatic bud (choices A, B, and C) does not rotate around the duodenum and
therefore is not the cause of annular pancreas.

The ventral pancreatic bud does not form from the first part of the duodenum (choice D) and
therefore does not rotate around this part of the duodenum.

A newborn infant has some of its abdominal viscera protruding through a defect in the abdominal wall.
Which of the following is the likely cause of this defect?

A. Failure of the intestinal loop to retract from the umbilical cord

B. Failure of the yolk stalk to degenerate

C. Failure of peritoneal fusion

D. Incomplete fusion of the lateral body folds

E. Umbilical herniation

Explanation:

The correct answer is D. During the fourth week of development, the lateral body folds move
ventrally and fuse in the midline to form the anterior body wall. Incomplete fusion results in
a defect that allows abdominal viscera to protrude from the abdominal cavity, a condition known
as gastroschisis.

During development, the midgut normally herniates into the umbilical cord and then subsequently
retracts into the abdominal cavity. Failure of the intestinal loop to retract from the
umbilical cord (choice A) results in omphalocele.

Failure of the yolk stalk to degenerate (choice B) results in an ileal (Meckel's) diverticulum
or a vitelline fistula or cyst. In the early embryo, the gut tube is connected to the yolk sac
by a narrow connection known as the yolk stalk. Normally, this connection degenerates.

During development, certain peritoneal organs fuse with the posterior abdominal wall to become
secondarily retroperitoneal. Failure of this peritoneal fusion (choice C) will result in
certain organs that are normally immobile being mobile (e.g., mobile cecum).

Umbilical herniation (choice E) results from abdominal viscera protruding through a weakness in
the abdominal wall after development. Such protrusions are covered by subcutaneous fascia and
skin, distinguishing them from gastroschisis.
CELL BIOLOGY

Autopsy of a baby who died minutes after birth demonstrates intestines within the thoracic cavity. Which
of the following most probably accounts for these findings?

A. Heterotopia

B. Hypertrophic pyloric stenosis

C. Large sliding hernia

D. Maldevelopment of the lungs

E. Partial absence of the diaphragm

Explanation:

The correct answer is E. In some infants, there is a weakness or partial-to-total absence of


the diaphragm (often on the left) which, if sufficiently large, can permit herniation of
abdominal contents into the thorax. Small defects are usually surgically correctable, but large
ones may cause rapidly lethal respiratory embarrassment in the newborn.

Heterotopia (choice A) is the presence of small amounts of normal tissue in abnormal sites.

Congenital hypertrophic pyloric stenosis (choice B) can cause persistent vomiting in infancy,
but does not cause herniation of intestines into the thorax.

Death in infancy due to intestinal herniation into the thorax is not related to sliding hernia
(choice C).

Maldevelopment of the lungs (choice D) is a result rather than the cause of this baby's problems.

During development, the formation of the kidney is induced by which of the following structures?

A. Allantois

B. Mesonephric duct

C. Mesonephros

D. Metanephric duct

E. Urogenital ridge

Explanation:

The correct answer is D. The metanephric duct (also known as the ureteric bud) is a
diverticulum of the mesonephric duct. It grows to the metanephric mass of the urogenital ridge.
It induces the development of the metanephros, which will give rise to the excretory units of
the definitive kidney. The metanephros, in turn, induces the metanephric duct to divide into
the calyces and the collecting tubules.

The allantois (choice A) is an endodermal diverticulum of the yolk sac. It becomes incorporated
into the urogenital sinus and contributes to the formation of the urinary bladder. It is not
involved with the formation of the kidney.

The mesonephric duct (choice B) is a mesodermal duct into which the mesonephric tubules drain. The
mesonephric duct is the same as the pronephric duct, but at a later stage of development. The
mesonephric duct develops into the ductus deferens in the male. The metanephric duct is a
diverticulum of the mesonephric duct.

The mesonephros (choice C) is the embryonic kidney, which functions in the embryo before the
metanephros (or definitive kidney) develops. The mesonephros develops from the urogenital ridge.
CELL BIOLOGY

The urogenital ridge (choice E) is a longitudinal elevation of the intermediate mesoderm. The
embryonic and adult kidneys, as well as the gonads, develop from this mesoderm.

Which of the following developmental abnormalities might account for unexplained small intestinal
bleeding?

A. Central nervous system heterotopia

B. Gastric heterotopia

C. Pancreatic heterotopia

D. Parathyroid heterotopia

E. Thyroid heterotopia

Explanation:

The correct answer is B. Heterotopic rests are small areas of normal tissue in abnormal sites.
These are usually clinically insignificant unless they form a noticeable mass or are
misdiagnosed in a biopsy (raising suspicion of metastatic cancer). One exception is a gastric
heterotopia, which typically occurs in the small intestine, and can produce enough acid to
cause a peptic ulcer in adjacent mucosa. The ulcer may be a source of gastrointestinal
bleeding.

Heterotopias of the CNS (choice A), parathyroid (choice D), and thyroid(choice E) can occur,
but would not be expected in the small intestine and would not be likely to cause bleeding if present.

Pancreatic heterotopia (choice C) can occur in the small intestine, but does not cause bleeding.

When examining a histological section of a normal ovary, a technician notices an oocyte surrounded by
several layers of follicular cells. A small antrum is present. Which of the following is the
correct term for the entire structure, composed of the oocyte, follicular cells, and antrum?

A. Corpus luteum

B. Graafian follicle

C. Primary follicle

D. Primordial follicle

E. Secondary follicle

Explanation:

The correct answer is E. Follicles in different stages of maturation have different


appearances. The most primitive follicles, primordial follicles (choice D), are inactive
reserve follicles that contain primary oocytes (arrested in prophase of first meiotic division)
surrounded by a single layer of flattened follicular cells. Primary follicles (choice C), the
next stage, are slightly larger and contain a central oocyte surrounded by one or several
cuboidal follicular cells. When several small spaces in the follicular mass fuse to form the
antrum (follicular cavity), the follicle is termed a secondary follicle (choice E). The
secondary follicles continue to enlarge, and develop a more complex structure that includes
cumulus oophorus, corona radiata, theca interna, theca externa, and zona pellucida. The
Graafian follicle (choice B) is the mature form of the follicle, which extends through the
entire cortex and bulges out at the ovarian surface. After it ruptures and releases the ovum,
the corpus luteum (choice A)develops as the cells of the follicle and the theca interna cells
enlarge, become epithelioid, and secrete estrogen. The granulosa lutein cells contain yellow
pigment and secrete progesterone. If pregnancy does not occur, the corpus luteum eventually
degenerates; if pregnancy occurs, it is maintained throughout the pregnancy.
CELL BIOLOGY

Which of the following cell types is derived from neuroepithelial cells?

A. Astrocytes

B. Enterochromaffin cells

C. Melanocytes

D. Odontoblasts

E. Schwann cells

Explanation:

The correct answer is A. Astrocytes and oligodendrocytes are both derived from glioblasts,
which, in turn, are derived from neuroepithelial cells. Other neuroepithelial cell derivatives
include neuroblasts and ependymal cells.

All the other choices are derived from neural crest cells. Other neural crest derivatives
include the neurons of the parasympathetic and sympathetic ganglia (including the adrenal
medulla), the dorsal root ganglia of the peripheral nervous system, the sensory ganglia of
cranial nerves V, VII, IX, and X, and the leptomeninges (pia and arachnoid).

Which of the following embryonic structures gives rise to the adrenal cortex?

A. Ectoderm

B. Endoderm

C. Mesoderm

D. Mesonephros

E. Neural crest cells

Explanation:

The correct answer is C. The mesoderm gives rise to the adrenal cortex. In addition, it also
gives rise to connective tissue, cartilage, bone, muscle, blood and lymph vessels, kidneys, gonads,
serous membranes lining body cavities, and the spleen.

The ectoderm (choice A) gives rise to the central nervous system, peripheral nervous system,
epidermis and its appendages, mammary glands, pituitary gland, tooth enamel, and the neural crest.

The endoderm (choice B) gives rise to the parenchyma of the tonsils, thyroid and parathyroid
glands, thymus, liver, pancreas, the epithelial lining of the GI and respiratory tracts,
urinary bladder, urethra, and auditory tube.

The mesonephros (choice D) functions as an interim kidney in the embryo.

The neural crest cells (choice E) give rise to cells of the spinal and cranial nerves,
autonomic ganglia, melanocytes, leptomeninges, connective tissue and bone of branchial arch
origin, and the adrenal medulla.

A neuroscientist is investigating the development of the nervous system. In his experiments, he


microinjects a dye into the embryo of an animal subject in vivo. After birth, he performs
histological studies to determine the destination of the dye. In one animal subject, he locates
the dye in the dorsal horn of the spinal cord. Which of the following locations in the embryo
was the most likely site of the injection?

A. Alar plate
CELL BIOLOGY

B. Basal plate

C. Neural crest

D. Rostral end of neural tube

E. Sulcus limitans

Explanation:

The correct answer is A. The spinal cord arises from the caudal end of the neural tube. During
development, an alar and a basal plate is formed, separated by a longitudinal groove called the
sulcus limitans (choice E). The alar plate forms the dorsal (posterior) part of the spinal cord
and becomes the sensory or afferent portion of the cord. The basal plate (choice B) is the
ventral (anterior) part of the cord, and becomes the motor, or efferent, portion of the spinal
cord, and therefore would contain anterior horn cells.

The neural crest (choice C) develops into multipolar ganglion cells of autonomic ganglia,
pseudounipolar cells of spinal and cranial nerve ganglia, leptomeningeal cells, Schwann cells,
melanocytes, chromaffin cells of the adrenal medulla, and odontoblasts.

The brain forms from the rostral end of the neural tube (choice D).

A newborn boy does not pass meconium until 48 hours after his birth. Two weeks later his mother reports
that he has not been passing stool regularly. Anorectal manometry reveals increased internal
anal sphincter pressure on rectal distention with a balloon. Radiographic studies reveal
massive dilation of the colon proximal to the rectum. This indicates a developmental
abnormality in which of the following embryonic tissues?

A. Ectoderm

B. Endoderm

C. Neural crest

D. Neural ectoderm

E. Splanchnic mesoderm

Explanation:

The correct answer is C. The baby has Hirschsprung's disease, which is due to an absence of
ganglion cells in the wall of the colon. Neural crest cells contribute to the formation of many
adult structures. Among these are all of the postganglionic neurons of the autonomic nervous
system and the sensory neurons of the peripheral nervous system.

Ectoderm (choice A) forms the epidermis of the skin and the parenchymal cells of glands
associated with the skin such as the sweat glands, sebaceous glands, and mammary glands.

Endoderm (choice B) forms the epithelial lining of the gut tube and the parenchymal cells of
glands associated with the gut tube, such as the liver and pancreas.

Neural ectoderm (choice D) forms the central nervous system, the somatic motor neurons of the
peripheral nervous system, and the preganglionic neurons of the autonomic nervous system.

Splanchnic mesoderm (choice E) forms the visceral peritoneum, visceral pleura, visceral
pericardium, and the stroma and muscle of the wall of the gut, among other structures.

Physical examination of a young boy reveals discharge of urine from the umbilicus. The physician
concludes that the urachus has failed to fuse. Which of the following structures is the normal
adult remnant of the fused urachus?
CELL BIOLOGY

A. Lateral umbilical fold

B. Medial umbilical fold

C. Medial umbilical ligament

D. Median umbilical fold

E. Median umbilical ligament

Explanation:

The correct answer is E. The urachus is the derivative of the allantoic duct, which passes
from the urogenital sinus to the umbilical cord. Normally, this duct fuses and is no longer
patent. The adult derivative is the median umbilical ligament, which lies in the midline along
the interior surface of the anterior abdominal wall. It passes from the upper end of the
bladder to the umbilicus.

The lateral umbilical fold (choice A) is the fold of parietal peritoneum that covers the
inferior epigastric artery and vein on the interior surface of the anterior abdominal wall.

The medial umbilical fold (choice B) is the fold of parietal peritoneum that covers the medial
umbilical ligament, the adult derivative of the umbilical artery (see below).

The medial umbilical ligament (choice C) is the adult derivative of the distal portion of the
umbilical artery. The umbilical artery arises from the internal iliac artery. It passes along
the bladder and then the anterior abdominal wall to reach the umbilicus. Prenatally, this
artery carries fetal blood to the placenta, where it gains oxygen and nutrients. Postnatally,
the proximal part of the umbilical artery remains patent and supplies blood to the superior
surface of the bladder. Distal to the bladder, the artery becomes fibrotic and is known as the
medial umbilical ligament.

The median umbilical fold (choice D) is the fold of parietal peritoneum that covers the median
umbilical ligament, the adult derivative of the urachus.

A male infant presents with flattened facial features, low set ears, and deformities of the feet. The
lungs are underdeveloped. The pregnancy was complicated by severe oligohydramnios. The most likely
cause
of this condition is a malformation of the

A. paramesonephric ducts

B. pronephros

C. ureteric buds

D. urogenital sinus

E. urorectal septum

Explanation:

The correct answer is C. This infant suffers from Potter syndrome, caused by an absence of both
kidneys. This is often caused by a failure of the ureteric buds to develop. The ureteric buds
form the ureters, renal pelvis, calyces, and collecting tubules. The collecting tubules induce
the formation of metanephric vesicles, which differentiate into the tubular components of the
nephron.

The paramesonephric ducts (choice A) form the uterine tubes, uterus, and the upper third of the
vagina.

The pronephros (choice B) is a transient collection of cells that disappears during the fourth week of
CELL BIOLOGY

development. It does not form the permanent kidney.

The urogenital sinus (choice D) forms the urinary bladder (in both sexes). It also forms the
prostatic urethra, prostate, membranous urethra, bulbourethral glands, penile urethra (proximal
part) and Littre's glands in the male. In the female it forms most of the urethra, urethral
glands, paraurethral glands, vestibule of the vagina and greater vestibular glands.

The urorectal septum (choice E) partitions the cloaca into the urogenital sinus and rectum (and
upper anal canal).

A 47-year-old man presents to his neurologist with an unsteady broad-based gait and slow, slurred
speech. Neurological examination reveals dysdiadochokinesis, intention tremor, hypotonia, and
nystagmus. The patient's lesion is in a brain structure that derives from which of the following
embryonic structures?

A. Diencephalon

B. Mesencephalon

C. Metencephalon

D. Myelencephalon

E. Telencephalon

Explanation:

The correct answer is C. The patient in question has a cerebellar lesion. Cerebellar
dysfunction can lead to a variety of motor dysfunctions, including truncal ataxia (appearing
similar to the gait of an intoxicated individual), intention tremor (uncontrolled shaking of
affected extremity present only with purposeful movement), dysdiadochokinesia (the inability to
perform rapid and regular alternating movements), dysmetria (inability to stop movements at the
desired point), dysarthria (ataxic speech), hypotonia, and nystagmus.

During the fourth week of embryonic development, the anterior end of the neural tube develops
three vesicles, the prosencephalon (forebrain), the mesencephalon (midbrain), and the
rhombencephalon (hindbrain). By the sixth week, five vesicles (listed in the answer options)
have developed. The rhombencephalon has now developed into the metencephalon and
myelencephalon. The cerebellum and pons derive from the metencephalon.

The diencephalon (choice A), which is derived from the prosencephalon, develops into the thalamus,
hypothalamus, epithalamus, subthalamus (everything with the word "thalamus"), posterior lobe of
the pituitary, and neural retina.

The mesencephalon (choice B), or midbrain, is the only brain vesicle that does not produce a secondary
vesicle; it remains the mesencephalon.

The myelencephalon (choice D), which is derived from the rhombencephalon, develops into the medulla
oblongata.

The telencephalon (choice E), which is derived from the prosencephalon, develops into the cerebral
hemispheres (cerebral cortex, basal ganglia, and deep white matter).

Evaluation of an infant with a variety of congenital abnormalities reveals hypocalcemia due to a lack of
parathyroid hormone. On x-ray, the thymic shadow is absent. A failure of development and
differentiation of which of the following embryonic structures would most likely be responsible
for the observed presentation?

A. Second pharyngeal arch

B. Second pharyngeal cleft


CELL BIOLOGY

C. Second pharyngeal pouch

D. Third pharyngeal arch

E. Third pharyngeal pouch

Explanation:

The correct answer is E. The third pharyngeal pouch normally gives rise to the inferior
parathyroid glands and the thymus. Cells from these pouches migrate caudally in the embryo to
the eventual location of these organs. The superior parathyroid glands are derived from the
fourth pharyngeal pouch. The absence of these organs is found in DiGeorge syndrome, which
typically presents with immunodeficiency and hypoparathyroidism.

The second pharyngeal arch (choice A) gives rise to several skeletal and muscular structures.
These include the stapes, styloid process, stylohyoid ligament, a portion of the hyoid bone,
the stapedius muscle, stylohyoid muscle, posterior digastric muscle, and all of the muscles of
facial expression.

The second pharyngeal cleft (choice B) is normally obliterated. A persistence of this cleft may
result in a lateral cervical cyst, sinus, or fistula.

The second pharyngeal pouch (choice C) gives rise to the tonsillar fossa and the palatine tonsils.

The third pharyngeal arch (choice D) gives rise to most of the hyoid bone and the
stylopharyngeus muscle. The third aortic arch, which passes through the third pharyngeal arch,
gives rise to the common carotid artery and the internal carotid artery.

Most of the oocytes in the ovary of a prepubescent girl are in which meiotic stage?

A. Anaphase of the second meiotic division

B. Metaphase of the first meiotic division

C. Metaphase of the second meiotic division

D. Prophase of the first meiotic division

E. Telophase of the first meiotic division

Explanation:

The correct answer is D. The first meiotic division is the "reduction" meiotic division, in
which the diploid complement of DNA is reduced to a haploid complement. The bulk of oocytes in
premenopausal women, girls, and babies are arrested at prophase of the first meiotic division.
Postmenopausal women have very few viable oocytes.

The heart of an embryo first begins beating at which of the following ages?

A. 2 weeks

B. 3 weeks

C. 4 weeks

D. 6 weeks

E. 8 weeks

Explanation:

The correct answer is C. While the third-week embryo is a primitive trilaminar plate, in the
CELL BIOLOGY

fourth week, the heart begins to form and begins beating almost immediately. Hematopoiesis
occurs in the yolk sac, and a primitive circulatory system connects the capillary plexuses of
the yolk sac and chorion to the embryo. Partitioning of the atrium also begins in the fourth
week. During the fifth week, cardiac septa form and the atrioventricular (AV) cushions fuse. By
the sixth week, the heart is close to fully formed. This early sequence for the heart explains
why it is so difficult to try to prevent congenital malformations of the cardiovascular system
from occurring, since the mother of a 6-week-old fetus is only about 8 weeks from her last
menstrual period, and may have assumed that she just "missed a period" (a very common
phenomenon) for reasons other than pregnancy.

A 17-year-old male is examined by a physician, who notes a mass at the back of the young man's tongue.
The physician biopsies the mass, and the pathology report comes back with a diagnosis of normal
thyroid tissue. The occasional presence of such tissue at the back of the tongue is related to
the embryonic origin of the thyroid near which of the following structures?

A. First pharyngeal pouch

B. Foramen cecum

C. Nasolacrimal duct

D. Second pharyngeal arch

E. Third pharyngeal pouch

Explanation:

The correct answer is B. The thyroid gland originates as a mass of endodermal tissue near the
foramen cecum, which is near the tuberculum impar (which becomes the central part of the
tongue). During development, the thyroid descends in front of the pharynx, maintaining a
connection to the tongue via the thyroglossal duct. Usually, the thyroglossal duct disappears.
Uncommonly, residual ectopic thyroid tissue can be left anywhere along the path, including at
the back of the tongue. (In rare patients, all of the thyroid tissue remains at this site,
forming a mass that should not be excised, for obvious reasons!)

The first pharyngeal pouch (choice A) develops into the middle ear and eustachian tube.

The nasolacrimal ducts (choice C) connect the eyes to the mouth.

The second pharyngeal arch (choice D) develops into many muscles of the face and styloid
process of the temporal bone.

The third pharyngeal pouch (choice E) develops into the thymus and inferior glands.

USMLE STEP1 MCQ STUDY ONLINE QUESTIONS REVIEW 4


(10)<1>A 68-year-old female who recently had a cholecystectomy develops a
fever of 103°F and has persistent drainage from her biliary catheter.
She is given cephalothin and gentamicin for 10 days. Her serum creatinine
level increases to 7.6 mg/dL. Her urine output is 1.3 L/day and has not
diminished over the past few days. There is no history of hypotension
and her vital signs are normal. Renal ultrasonography shows no evidence
of obstruction. The most likely etiology of the patient's condition is
A. acute glomerulonephritis
B. acute renal failure secondary to cephalothin
C. gentamicin nephrotoxicity
D. renal artery occlusion
E. sepsis
Answer
CELL BIOLOGY

--------------------------------------------------------------------------------
<2>A 45-year-old man presents to a physician with back pain and facial pain.
Physical examination demonstrates coarse facial features and kyphosis.
Laboratory examination is remarkable for elevated alkaline phosphatase
. X-ray studies demonstrate skull thickening with narrowing of foramina,
and bowing of the femur and tibia. Bone biopsy reveals a mosaic
pattern of bone spicules with prominent osteoid seams. Which of
the following neoplasms occurs at an increased frequency in patients
with this disorder?
A. Astrocytoma
B. Hodgkin's lymphoma
C. Meningioma
D. Non-Hodgkin's lymphoma
E. Osteosarcoma
Answer
--------------------------------------------------------------------------------
<3> Which of the labeled structures is responsible for the secretion
of renin?
A. A
B. B
C. C
D. D
E. E
Answer

--------------------------------------------------------------------------------

<4>A 42-year-old African-American man sustains severe injuries in an


automobile accident and is admitted to the intensive care unit.
Examination of a peripheral blood smear on the 3rd day of admission
reveals helmet cells, schistocytes, and decreased platelets. Which of
the following is most strongly suggested by these findings?
A. Autoimmune hemolysis
B. Disseminated intravascular coagulation (DIC)
C. Hereditary spherocytosis
D. Megaloblastic anemia
E. Sickle cell anemia
Answer
--------------------------------------------------------------------------------
<5>A 12-month-old child is diagnosed with an atrial septal defect.
What is the most common cause of such a congenital heart malformation?
A. Failure of formation of the septum primum
B. Failure of formation of the septum secundum
C. Incomplete adhesion between the septum primum and septum secundum
D. Malformation of the membranous interventricular septum
E. Malformation of the muscular interventricular septum
Answer
--------------------------------------------------------------------------------
<6>A 72-year-old man with prostate cancer is treated with leuprolide.
What is the mechanism of action of this drug?
A. It inhibits 5a-reductase
B. It is a competitive antagonist at androgen receptors
C. It is a competitive inhibitor of LH
D. It is a synthetic analog of GnRH
CELL BIOLOGY

E. It is a testosterone agonist
Answer
--------------------------------------------------------------------------------

<7>An XX genotypic infant is born with ambiguous genitalia. Laboratory


examination reveals hypoglycemia, hyperkalemia, and salt wasting. S
erum 17-OH progesterone is markedly increased. Which of the following is
the most likely diagnosis?
A. 5-alpha-reductase deficiency
B. 11-beta-hydroxylase deficiency
C. 17-alpha-hydroxylase deficiency
D. 21-hydroxylase deficiency
E. Complete androgen resistance
Answer
--------------------------------------------------------------------------------

<8>A 57-year-old fisherman with a history of alcoholism is hospitalized in


Gulfport, Mississippi with a 1-day history of severe, watery diarrhea after
eating several raw oysters. He is badly dehydrated on admission, and within
12 hours, he becomes severely hypotensive and dies. Which of the following
pathogens is the most likely cause of this man's death?
A. Citrobacter diversus
B. Enterotoxigenic E. coli
C. Providencia stuartii
D. Vibrio cholerae
E. Vibrio vulnificus
Answer
--------------------------------------------------------------------------------

<9>An Hispanic male is referred to the dermatology clinic of a major medical


center. On physical examination, the man has several disfiguring lesions on
his face and there is loss of cutaneous sensation to fine touch, pain, and
temperature. An acid-fast organism is observed in scrapings from a skin
lesion. Which of the following organisms is the most likely cause of this
patient's disease?
A. Bartonella henselae
B. Listeria monocytogenes
C. Mycobacterium avium-intracellulare
D. Mycobacterium leprae
E. Nocardia asteroides
Answer
--------------------------------------------------------------------------------

<10>Microscopic examination of a section of a normal young adult ovary


reveals large numbers of unusually large cells surrounded by a single
layer of flat epithelial cells. In which phase of the cell cycle are
these cells arrested?
A. Diplotene stage of the 1st meiotic division
B. Diplotene stage of the 2nd meiotic division
C. Metaphase stage of mitosis
D. Prophase stage of mitosis
E. Prophase stage of the 2nd meiotic division
Answer
--------------------------------------------------------------------------------
CELL BIOLOGY

<11>A 3-year-old child is seen by a pediatrician because he has developed


multiple isolated lesions on his face and neck. Physical examination
reveals many lesions up to 4 cm in diameter with golden crusts, while
in other sites small blisters and weeping areas are seen. Which of the
following is the most likely diagnosis?
A. Aphthous ulcers
B. Erysipelas
C. Herpes simplex I
D. Impetigo
E. Measles
Answer
--------------------------------------------------------------------------------

<12>A 2-year-old-boy is brought to the pediatrician by his mother because


he has had several episodes of rectal bleeding. Evaluation with a
technetium-99m perfusion scan reveals a 3-cm ileal outpouching located
60 cm from the ileocecal valve. This structure likely contains which of
the following types of ectopic tissue?
A. Duodenal
B. Esophageal
C. Gastric
D. Hepatic
E. Jejunal
Answer
--------------------------------------------------------------------------------

<13>A 19-year-old college student presents to the student health clinic


complaining of weakness, malaise, and a chronic cough. He has a fever of
100 degrees F and a dry cough; no sputum can be obtained for laboratory
analysis, so a bronchial lavage is performed and the washings are
submitted to the laboratory. The laboratory reports that the organism
is "slow-growing." Serodiagnosis reveals Strep MG agglutinins in the
patient's serum. Which of the following organisms is the most likely
cause of this student's illness?
A. Klebsiella pneumoniae
B. Mycoplasma pneumoniae
C. Parainfluenza virus
D. Respiratory syncytial virus
E. Streptococcus pneumoniae
Answer
--------------------------------------------------------------------------------

<14>A 52-year-old male is brought into the emergency room by his wife
because he has been complaining of a severe headache. Physical exam
reveals ptosis of the right eyelid with the right eye facing down and out.
There is a fixed and dilated right pupil with an inability to accommodate.
Subarachnoid blood appears on noncontrast CT scan. Magnetic resonance
angiography (MRA) would be expected to reveal an aneurysm of which of
the following vessels?
A. Anterior cerebral artery
B. Anterior choroidal artery
C. Anterior communicating artery
D. Middle cerebral artery
CELL BIOLOGY

E. Ophthalmic artery
F. Posterior communicating artery
G. Posterior inferior cerebellar artery
Answer
--------------------------------------------------------------------------------

<15>A 24-year-old graduate student presents to a physician with complaints


of severe muscle cramps and weakness with even mild exercise. Muscle
biopsy demonstrates glycogen accumulation, but hepatic biopsy is
unremarkable. Which of the following is the most likely diagnosis?
A. Hartnup's disease
B. Krabbe's disease
C. McArdle's disease
D. Niemann-Pick disease
E. Von Gierke's disease
Answer

--------------------------------------------------------------------------------

<16>A baby is born with a flat facial profile, prominent epicanthal folds,
and simian crease. She vomits when fed, and upper GI studies demonstrate
a "double bubble" in the upper abdomen. Which of the following
cardiovascular abnormalities might this child also have?
A. Atrial septal defect
B. Berry aneurysm
C. Coarctation of the aorta
D. Endocardial cushion defect
E. Tetralogy of Fallot
Answer
--------------------------------------------------------------------------------

<17>An asymptomatic, 24-year-old African-American woman in her second


trimester of pregnancy has the following laboratory findings: Based
on the laboratory data, which of the following tests is necessary for fur
ther evaluation of this patient?
A. Creatinine clearance
B. Oral glucose tolerance test
C. Serum ferritin
D. Sickle cell preparation
E. No further study is necessary
Answer
--------------------------------------------------------------------------------

<18>Endometrial biopsy demonstrates a thick endometrium with long,


coiled glands lined by a columnar epithelium with prominent cytoplasmic
vacuoles adjacent to the gland lumen. Earlier in the menstrual cycle,
the glands were much smaller and were lined with cells that did not
have vacuoles. Which of the following hormones is primarily responsible
for inducing this change in appearance?
A. Aldosterone
B. Cortisol
C. Estrogen
D. Progesterone
E. Thyroxine
CELL BIOLOGY

Answer
--------------------------------------------------------------------------------

<19>A 28-year-old man discovers a mass in his neck while buttoning


his shirt collar. Physical examination reveals a 2 cm mass in one
thyroid lobe, which is "cold" on scintiscan. Aspiration of the
nodule demonstrates small "solid balls" of neoplastic follicular
cells. Careful examination of these tissue balls reveals that
they contain microscopic blood vessels and fibrous stroma in
their centers. Which of the following is the most likely diagnosis?
A. Follicular carcinoma
B. Hashimoto's disease
C. Medullary carcinoma
D. Papillary carcinoma
E. Thyroid adenoma
Answer
--------------------------------------------------------------------------------

<20>Nissl bodies correspond to which of the following cytoplasmic organelles?


A. Golgi apparatus
B. Mitochondria
C. Nucleoli
D. Rough endoplasmic reticulum
E. Smooth endoplasmic reticulum
Answer
--------------------------------------------------------------------------------

<21>A 40-year-old man with sleep apnea participates in a sleep study.


During his evaluation, normal sawtooth waves are observed on his EEG
tracing. This pattern is associated with which period of sleep?
A. REM
B. Stage 1
C. Stage 2
D. Stage 3
E. Stage 4
Answer
--------------------------------------------------------------------------------

<22>A United Nations representative from a poor African country has an


influenza-like illness that resolves in a few days. Less than a week later,
however, he develops muscle pain, spasms, and sensory disturbances.
Two days after this, flaccid paralysis occurs. Which of the following
is most likely to be immediately life-threatening in this patient?
A. Acute renal failure
B. Bowel paralysis
C. Fulminant liver failure
D. Gastrointestinal bleeding
E. Respiratory paralysis
Answer
--------------------------------------------------------------------------------

<23>What is the normal myeloid to erythroid ratio in bone marrow?


A. 1:1
CELL BIOLOGY

B. 1:3
C. 1:10
D. 3:1
E. 10:1
Answer
--------------------------------------------------------------------------------

<24>A 45-year-old male complains of fatigue and increased frequency of


urination. Questioning reveals that he is somewhat confused. Physical
examination reveals a 5'10", 240 lb. individual whose fat is centrally
distributed. Urine is positive for glucose, but negative for ketones.
A blood sample drawn from this patient is likely to exhibit which of
the following compared to that of a normal individual?
A. Decreased concentration of C-peptide
B. Decreased pH
C. Increased antibodies against islet cell proteins
D. Increased concentration of b-hydroxybutyrate
E. Increased osmolarity
Answer
--------------------------------------------------------------------------------

<25>At which of the following ages does fetal movement first occur?
A. 1 month
B. 2 months
C. 4 months
D. 6 months
E. 7 months
Answer
--------------------------------------------------------------------------------

<26>A child with retinoblastoma is found to have a 13q14 deletion.


The Rb gene, which resides at this locus, produces which kind of
tumor-associated protein?
A. Cell cycle regulator
B. Growth factor
C. Growth factor-binding protein
D. Growth factor receptor
E. Transcription activator
Answer
--------------------------------------------------------------------------------

<27>A 29-year-old woman is involved in an automobile accident, and


is taken to the emergency room by paramedics. X-rays reveal a
fracture of her pelvis. During the healing of the pelvic fracture,
the obturator nerve becomes entrapped in the bone callus. Compression
of this nerve by the growing bone may result in weakness of which of
the following muscles?
A. Adductor magnus
B. Biceps femoris
C. Rectus femoris
D. Sartorius
E. Vastus medialis
Answer
--------------------------------------------------------------------------------
CELL BIOLOGY

<28>An African child develops massive unilateral enlargement of his lower


face in the vicinity of the mandible. Biopsy demonstrates sheets of
medium-sized blast cells with admixed larger macrophages. This
type of tumor has been associated with which of the following?
A. Epstein-Barr virus and t(8;14)
B. Hepatitis B and t(9;22)
C. Herpesvirus and CD5
D. Human immunodeficiency virus and CD4
E. Human papillomavirus and t(2;5)
Answer
--------------------------------------------------------------------------------

<29>A 57-year-old man presents for a routine physical. His blood pressure
is 161/98 mm Hg. The patient's only complaint is that over the past
several months he has had difficulty urinating. His urine stream is
intermittent, and he has recently begun experiencing nocturia and
profound urinary urgency. Digital rectal exam reveals diffuse
enlargement of the prostate. Which of the following agents
would be most likely to effectively treat the man's urinary
tract symptoms as well as his hypertension?
A. Finasteride
B. Guanfacine
C. Hydralazine
D. Labetalol
E. Terazosin
Answer
--------------------------------------------------------------------------------
Answers
--------------------------------------------------------------------------------

1The correct answer is C. A small percentage of patients (5% to 10%) develop a nonoliguric form of acute renal
failure when treated with aminoglycosides such as gentamicin. Gentamicin can accumulate in the kidney to produce a
delayed form of acute renal failure resulting in an elevation of the serum creatinine level. The nonoliguric form of
renal failure, seen in this patient, is the typical presentation for gentamicin nephrotoxicity.
Acute glomerulonephritis (choice A) is typically associated with hypertension and the appearance of an active urinary
sediment containing casts and red blood cells.
Cephalothin (choice B) is a first-generation cephalosporin commonly used in the treatment of severe infection of the
genitourinary tract, gastrointestinal tract, and respiratory tract, as well as skin infections. This antibiotic can produce
an acute interstitial nephritis; however, the patient's presentation is consistent with gentamicin nephrotoxicity.
Interstitial nephritis is commonly associated with the development of acute renal failure, fever, rash, and
eosinophilia.
Renal artery occlusion (choice D) is commonly caused by thrombosis or embolism. The clinical features of acute renal
artery occlusion are hematuria, flank pain, fever, nausea, elevated LDH, elevated SGOT and acute renal failure.
Since the patient has normal vital signs and no history of hypotension, a diagnosis of sepsis (choice E) is unlikely.
--------------------------------------------------------------------------------2The correct answer is E. The phrase "mosaic
pattern" of newly formed woven bone is a specific tip-off for Paget's disease of bone, and is not seen in other bone
conditions. The clinical and radiologic presentation are typical; an increased hat size may also be a clue. In its early
stages, Paget's disease is characterized by osteolysis, producing patchwork areas of bone resorption with bizarre,
large osteoclasts. In the middle stage of the disease, secondary osteoblastic activity compensates with new bone
formation, producing the mosaic pattern. In late Paget's, the bones are dense and osteosclerotic. Paget's disease is
suspected to be related to prior viral infection, but the cause remains mysterious. Complications include
myelophthisic anemia, high output cardiac failure, pain secondary to nerve compression, deformities secondary to
skeletal changes, and in about 1% of patients, osteosarcoma or other sarcoma, typically involving the jaw, pelvis, or
CELL BIOLOGY

femur.
An increased incidence of astrocytomas (choice A) is associated with tuberous sclerosis.
Hodgkin's lymphoma (choice B) is usually a disease of young adults, although older patients may have the
lymphocyte-depleted form.
Meningiomas (choice C) are mostly benign tumors that affect adults, especially females. There may be an association
with breast cancer, possibly related to high estrogen states.
Non-Hodgkin's lymphoma (choice D) is more common in AIDS and other immunodeficiency states, although the
incidence in the immunocompetent is increasing.
--------------------------------------------------------------------------------3The correct answer is C. This question required
that you know two pieces of information: (1) renin is secreted from the juxtaglomerular (JG) cells; and (2) the
location of the JG cells in a picture that includes both the glomerulus and the juxtaglomerular apparatus. Remember
that renin is responsible for converting angiotensinogen to angiotensin I, which is subsequently converted to
angiotensin II in the lungs by an important enzyme appropriately called angiotensin-converting enzyme. Angiotensin
II helps preserve blood pressure both by its potent vasoconstrictor properties and by its ability to stimulate
aldosterone secretion from the adrenal cortex.
The label for choice A points to the glomerular basement membrane.
The label for choice B points to the epithelium of Bowman's capsule.
The label for choice D points to the macula densa cells, which are thought to sense sodium concentration in the
distal convoluted tubule.
The label for choice E points to the Polkissen cells. Their function is unknown.
--------------------------------------------------------------------------------4The correct answer is B. The findings suggest
disseminated intravascular coagulation (DIC), which is a feared complication of many other disorders, such as
obstetrical catastrophes, metastatic cancer, massive trauma, and bacterial sepsis. The basic defect in DIC is a
coagulopathy characterized by bleeding from mucosal surfaces, thrombocytopenia, prolonged PT and PTT, decreased
fibrinogen level, and elevated fibrin split products. Helmet cells and schistocytes (fragmented red blood cells) are
seen on peripheral blood smear.
Autoimmune hemolysis (choice A) and hereditary spherocytosis (choice C) would be characterized by spherocytes in
the peripheral smear.
Macro-ovalocytes and hypersegmented neutrophils can be seen in megaloblastic anemia (choice D).
Sickle cells are seen in sickle cell anemia (choice E).
--------------------------------------------------------------------------------5The correct answer is C. The most common form
of atrial septal defect is located near the foramen ovale (not to be confused with a patent foramen ovale, which is of
little or no hemodynamic significance). They result from incomplete adhesion between the septum primum and the
septum secundum during development.
Atrial septal defects less commonly result from failures of formation of the septum primum (choice A) and septum
secundum (choice B).
Malformations of the interventricular septum (choices D and E) cause ventricular septal defects rather than atrial
septal defects.
--------------------------------------------------------------------------------6The correct answer is D. Leuprolide is a GnRH
analog. Given long-term in a continuous fashion, it will inhibit FSH and LH release, thereby decreasing testosterone
production and exacting a chemical castration in men. It can be used in the treatment of prostate cancer, polycystic
ovary syndrome, uterine fibroids, and endometriosis.
Inhibition of 5a-reductase (choice A) is the mechanism of action of finasteride. It thereby inhibits the production of
dihydrotestosterone. It is used in the treatment of benign prostatic hyperplasia (BPH).
Flutamide is another drug used in the treatment of prostate cancer. It is a competitive antagonist at androgen
receptors (choice B).
Since LH activates interstitial cells to secrete testosterone, a synthetic analog of LH (choice C) would not be
appropriate treatment for prostatic cancer. The same goes for a testosterone analog (choice E).
Here is a brief chart that will aid you in remembering the actions of these similar sounding drugs:
Drug Action Indication
Leuprolide GnRh analog Prostate CA
Flutamide Competitive androgen antagonist Prostate CA
Finasteride 5a-reductase inhibitor BPH
Remember, "loo"prolide and "floo"tamide are both used for prostate cancer. Finasteride is used for BPH.
CELL BIOLOGY

--------------------------------------------------------------------------------
7The correct answer is D. 21-hydroxylase deficiency is the most common form of congenital adrenal hyperplasia. The
simple virilizing variant (without salt wasting) is most common, but with severe 21-hydroxylase deficiency, virilization
and salt wasting occur. The infant described above exhibits salt wasting and hyperkalemia because aldosterone
secretion is diminished by the enzyme deficiency. The hypoglycemia is due to cortisol deficiency. Because cortisol
secretion is diminished in congenital adrenal hyperplasia, ACTH secretion from the anterior pituitary is increased due
to loss of negative feedback inhibition. The high levels of ACTH are responsible for the adrenal hyperplasia and the
increased secretion of the adrenal androgens, dehydroepiandrosterone and androstenedione, which are responsible
for the virilization of the external genitalia. 17-OH progesterone is the steroid precursor just proximal to 21-
hydroxylase and is also increased because of the excessive drive to the adrenal cortex by ACTH.
5-alpha-reductase deficiency (choice A) in male fetuses will produce normal differentiation of the internal
reproductive tracts, but the external genitalia will be feminized. This is because testosterone needs to be converted
to dihydrotestosterone (by 5-alpha-reductase) in the external genitalia and the prostate for normal differentiation
into the male phenotype.
11-beta-hydroxylase deficiency (choice B) is another form of congenital adrenal hyperplasia. It is characterized by
salt retention due to excessive secretion by the inner zones of the adrenal cortex of the weak mineralocorticoid,
deoxycorticosterone. Again, the excessive drive to the adrenal cortex is due to increased ACTH resulting from
diminished negative feedback suppression by cortisol. The adrenal also secretes excessive androgens and virilization
occurs in female fetuses.
17-alpha-hydroxylase deficiency (choice C) is another from of congenital adrenal hyperplasia that is accompanied by
salt retention. The high levels of ACTH drive the adrenal cortex to secrete increased amounts of deoxycorticosterone
and corticosterone, both of which have weak mineralocorticoid activity. Without the ability to 17-alpha-hydroxylate
progesterone or pregnenolone, steroid-secreting cells cannot produce sex steroids. When 17-alpha-hydroxylase
deficiency is present in the adrenal cortex, it is also present in the gonads. Hence, whether it occurs in a male or
female fetus, sex steroid production will be diminished. Female fetuses will develop normal reproductive tracts and
genitalia since these structures are programmed in utero to "automatically" become female. Male fetuses, however,
will have their reproductive tracts and genitalia feminized.
Complete androgen resistance (choice E) results in feminization of affected male fetuses. It is characterized by an XY
genotypic male with phenotypically female external genitalia and a vagina that ends as a blind sac.
--------------------------------------------------------------------------------8The correct answer is E. Vibrio vulnificus is an
extremely invasive organism, producing a septicemia in patients after eating raw shellfish, or causing wound
infections, cellulitis, fasciitis, and myositis after exposure to seawater or after cleaning shellfish. Patients at high risk
for septicemia include those with liver disease, congestive heart failure, diabetes mellitus, renal failure,
hemochromatosis, and immunosuppression.
Citrobacter diversus (choice A) produces neonatal meningitis and can be frequently cultured from the umbilicus.
Enterotoxigenic E. coli (choice B) produces the classic traveler's diarrhea. The toxin is ingested in water and salads.
The incubation period is approximately 12 hours. The diarrhea is non-inflammatory and treatment is supportive.
Providencia stuartii (choice C) is a gram-negative rod related to Proteus. It is a common cause of nosocomial
bacteremia in nursing home patients with chronic catheterization.
Vibrio cholerae (choice D) produces a non-invasive, non-inflammatory, high-volume secretory diarrhea that is toxin-
mediated.
--------------------------------------------------------------------------------9The correct answer is D. The disease in question
is leprosy, or Hansen's disease. A key feature in the description is the fact that the organism is acid-fast. Both of the
mycobacteria, M. avium-intracellulare and M. leprae are strongly acid-fast, that is they retain the carbol fuchsin dye
in the face of acid-alcohol decolorization. M. leprae has a predilection for the skin and cutaneous nerves, thereby
producing the symptoms of depigmentation and anesthetic cutaneous lesions. This loss of peripheral nerve function
leads to many of the disfiguring features of the disease; because the patients do not have normal pain sensation,
they sustain repeated injuries. In addition, the organism attacks cartilage and causes granuloma formation in the
skin, leading to some of the facial disfigurement.
Bartonella henselae (choice A) is a very small, gram-negative bacterium that is closely related to the rickettsia,
although it is able to be cultured on lifeless media. It is the cause of cat-scratch disease, a local, chronic
lymphadenitis most commonly seen in children, and bacillary angiomatosis, a disease seen particularly in AIDS
patients.
Listeria monocytogenes (choice B) is a ubiquitous microbe that causes disease in over 100 animal species. Although it
CELL BIOLOGY

is best known as an agent of meningitis in the newborn, it is a cause of multiple other diseases. A characteristic
feature of these infections is the development of granulomas at the site of the infection. The organism is not acid-
fast and has no particular predilection for skin or nervous tissues.
M. avium-intracellulare (choice C) causes tuberculosis-like pulmonary disease in the immunosuppressed.
Nocardia asteroides (choice E) primarily produces pulmonary infections in humans. The organism is consider to be
"weakly" acid-fast, meaning that if the amount of HCl used in the decolorization step is reduced, the organisms will
retain the carbolfuchsin primary stain.
--------------------------------------------------------------------------------10The correct answer is A. The cells described are
the primordial eggs, which remain stopped in the diplotene stage of the first meiotic division from before birth until
fertilization, a period which may be 40 or more years.
Choices B and E are incorrect because the oocytes are stopped in the first, not second meiotic division.
Choices C and D are incorrect because the cells described are oocytes and are not in mitosis.
--------------------------------------------------------------------------------11The correct answer is D. This is impetigo, which
is typically seen in preschool children with poor hygiene, particularly in the summer in warm climates. The
characteristic lesion has a large golden crust. Most cases are caused by Staphylococcus aureus; Streptococcus
pyogenes is occasionally implicated. Impetigo is highly infectious, and mini-epidemics can occur in daycare settings.
The initial treatment is typically with penicillins and topical preparations. Methicillin-resistant strains are presently rare
in this setting, but can occur.
Aphthous ulcers (choice A), commonly known as "canker sores," are painful, shallow ulcers of the oral cavity.
Erysipelas (choice B) is a different type of skin infection, often caused by Streptococcus pyogenes (also sometimes
others including Staphylococcus), and is characterized by large erythematous patches.
Herpes simplex I (choice C) causes tiny oral and perioral vesicles, but not large golden crusts.
Measles (choice E) causes a blotchy erythematous rash.
--------------------------------------------------------------------------------12The correct answer is C. This little boy has a
Meckel's diverticulum, an ileal outpocketing typically located within 50-75 cm of the ileocecal valve. It is a congenital
anomaly resulting from the persistence of the vitelline (omphalomesenteric) duct. Approximately half cause
ulceration, inflammation, and gastrointestinal bleeding due to the presence of ectopic acid-secreting gastric
epithelium. Pancreatic tissue may sometimes occur in these diverticula as well. Note that this is the most common
type of congenital gastrointestinal anomaly.
Something else to keep in mind: A favorite question attendings ask on the wards is the rule of 2's associated with
Meckel's diverticulum: it occurs in about 2% of children, occurs within approximately 2 feet of the ileocecal valve,
contains 2 types of ectopic mucosa (gastric and pancreatic), and its symptoms usually occur by age 2.
All of the other answer choices have no relationship to Meckel's diverticulum.
--------------------------------------------------------------------------------13The correct answer is B. The patient has primary
atypical pneumonia caused by Mycoplasma pneumoniae. This organism is fastidious and difficult to culture in the
laboratory, however serodiagnosis can be most helpful. Patients typically produce one or two heterophile antibodies
during the course of the infection; one agglutinates human O+ RBCs in the cold (the cold hemagglutinin) while the
other causes the agglutination of a strain of Streptococcus salivarius termed strain MG (the Strep MG agglutinins).
Klebsiella pneumoniae (choice A) is readily cultured on routine laboratory media and characteristically produces
pneumonia with blood clots in the sputum (red currant jelly sputum), which may be indicative of pulmonary abscess
development.
Parainfluenza viruses (choice C) cause croup, which is characterized by a dry, "barking" cough. It is more of a
tracheitis, bronchitis, and bronchiolitis than a pneumonitis. No heterophile antibodies are produced in these patients.
Respiratory syncytial virus (choice D) causes an atypical pneumonitis in infants. It is usually diagnosed by the
observation of syncytial masses in respiratory secretions. Cold hemagglutinins and Strep MG agglutinins are absent.
Streptococcus pneumoniae (choice E) is the number one cause of pneumonia in adults. It also causes septicemia and
meningitis in the elderly. The patient has a classical acute pneumonia with a productive cough, high fever with chills,
leukocytosis, tachycardia, rapid respirations and other signs of serious respiratory disease. A vaccine, composed of
the capsular carbohydrate of 23 serotypes of this organism, is routinely given to individuals over the age of 60, as
well as to individuals with splenic abnormalities (e.g., sickle cell disease) who are at increased risk for pneumococcal
sepsis.
--------------------------------------------------------------------------------14The correct answer is F. Aneurysm of the
posterior communicating artery is the second most common aneurysm of the circle of Willis (anterior communicating
artery is most common) and can result in third cranial nerve palsy (paralysis). The oculomotor nerve (CN III)
CELL BIOLOGY

innervates the levator palpebrae muscle. CN III paralysis would therefore result in ptosis (drooping of the upper
eyelid). CN III also innervates all of the extraocular muscles, except for the superior oblique (CN IV) and the lateral
rectus muscles (CNVI). Thus, CN III palsy would result in unopposed action of the superior oblique and lateral rectus
muscles, causing the affected eye to look down and out. CN III also supplies parasympathetic innervation to the
sphincter muscle of the iris (which constricts the pupil) and to the ciliary muscle. Interruption of this pathway leads
to a dilated and fixed pupil and to paralysis of accommodation.
Note that this question teaches you about another Boards-favorite pathology: subarachnoid hemorrhage (SAH). (In
this case, it was due to rupture of a posterior communicating artery aneurysm). A classic clue to the diagnosis is a
patient presenting with "the worst headache of their life." When you are presented a case of sudden severe
headache, SAH should rank highly on your differential diagnosis list.
The anterior cerebral artery (choice A) supplies the medial surface of the cerebral hemisphere, from the frontal pole
to the parieto-occipital sulcus. Occlusion may produce hypesthesia and paresis of the contralateral lower extremity.
The anterior choroidal artery (choice B) arises from the internal carotid artery and is not part of the circle of Willis. It
perfuses the lateral ventricular choroid plexus, the hippocampus, parts of the globus pallidus and posterior limb of
the internal capsule.
The anterior communicating artery (choice C) connects the two anterior cerebral arteries. It is the most common site
of aneurysm in the circle of Willis and may cause aphasia, abulia (impaired initiative), and hemiparesis.
The middle cerebral artery (choice D) supplies the lateral convexity of the cerebral hemisphere, including Broca's and
Wernicke's speech areas and the face and arm areas of the motor and sensory cortices. It also gives rise to the
lateral striate arteries, which supply the internal capsule, caudate, putamen, and globus pallidus. The middle cerebral
artery is the most common site of stroke.
The ophthalmic artery (choice E) enters the orbit with the optic nerve (CN II) and gives rise to the central artery of
the retina. Occlusion results in blindness.
The posterior inferior cerebellar artery (choice G) supplies the dorsolateral medulla and the inferior surface of the
cerebellar vermis. Occlusion may result in Wallenberg's syndrome: cerebellar ataxia, hypotonia, loss of pain and
temperature sensation of the ipsilateral face, absence of corneal reflex ipsilaterally, contralateral loss of pain and
temperature sensation in the limbs and trunk, nystagmus, ipsilateral Horner's syndrome, dysphagia, and dysphonia.
--------------------------------------------------------------------------------15The correct answer is C. A variety of glycogen
storage diseases exist, corresponding to defects in different enzymes in glycogen metabolism; most of these involve
the liver. McArdle's disease (Type V glycogen storage disease), due to a defect in muscle phosphorylase, is restricted
to skeletal muscle. The presentation described in the question stem is typical. Many affected individuals also
experience myoglobinuria. Definitive diagnosis is based on demonstration of myophosphorylase deficiency.
Hartnup's disease (choice A) is a disorder of amino acid transport.
Krabbe's disease (choice B) is a lysosomal storage disease.
Niemann-Pick disease (choice D) is a lysosomal storage disease.
Von Gierke's disease (choice E) is a glycogen storage disease with prominent involvement of liver, intestine, and
kidney.
--------------------------------------------------------------------------------16The correct answer is D. The disease is Down
syndrome (trisomy 21). In addition to mental retardation and the characteristic physical findings described in the
question stem, duodenal atresia is fairly common, as evidenced by the "double bubble" sign on x-ray. These children
are also likely to have various cardiac anomalies; endocardial cushion defect is the most common.
Atrial septal defect (choice A) is one of the most common genetic defects in the general population, but is less
common than endocardial cushion defect in patients with Down syndrome.
Berry aneurysms (choice B), also known as saccular aneurysms, are typically located in the circle of Willis on the
ventral surface of the brain. They occur more frequently in patients with adult polycystic disease. Rupture can
produce subarachnoid hemorrhage.
Coarctation of the aorta (choice C) occurs more commonly in females with a 45, XO genotype (Turner syndrome).
Tetralogy of Fallot (choice E) is the most common cause of early cyanosis, consisting of a ventricular septal defect,
right ventricular outflow tract obstruction, an overriding aorta, and right ventricular hypertrophy.
--------------------------------------------------------------------------------17The correct answer is E. All of the laboratory
data in this pregnant woman are normal, hence no further study is necessary. In a normal pregnancy, both the
plasma volume and RBC mass are increased with a greater increase in the plasma volume than RBC mass (2:1 ratio).
This has a dilutional effect on many laboratory tests.
Increasing plasma volume in pregnancy increases the creatinine clearance (choice A) due to the expected elevation
CELL BIOLOGY

in the glomerular filtration rate (GFR). The reference intervals for serum blood urea nitrogen and creatinine are lower
than normal, due to the dilutional effect of increased plasma volume and increased clearance of both analytes in the
urine caused by the rise in the GFR.
The threshold for glucose is reduced in pregnancy, so patients can have a positive dipstick test for glucose in the
presence of a normal serum glucose. Therefore, an oral glucose tolerance test (choice B) is not indicated.
The hemoglobin (Hb) concentration in pregnancy is normally decreased because of the dilutional effect of increased
plasma volume. Since the Hb is normal (for a pregnant woman) in this patient, a serum ferritin (choice C) to rule out
iron deficiency is unnecessary. Furthermore, iron deficiency is usually associated with a low MCV (microcytic anemia),
and her MCV is normal.
Although sickle disease is the most common genetic hemoglobinopathy among African Americans, the patient is not
anemic, so there is no reason to order a sickle cell preparation (choice D).
--------------------------------------------------------------------------------18The correct answer is D. The endometrial phase
with small glands is the proliferative phase; the one with large glands with secretory cells is the secretory phase.
Estrogen (choice C) is necessary for both phases, but it is the addition of progesterone (choice D), secreted by the
corpus luteum after the Graafian follicle ruptures, that triggers the switch from proliferative to secretory
endometrium.
Glucocorticoids (choice B) and the mineralocorticoid aldosterone (choice A) are secreted by the adrenal glands. They
do not produce the endometrial changes described.
Thyroxine (choice E) is secreted by the thyroid gland, and is unrelated to the observed morphologic changes in the
endometrium.
--------------------------------------------------------------------------------19The correct answer is D. The distinctive cell balls
described are broken-off papillary clusters, and are considered pathognomic for papillary carcinoma of the thyroid.
This is the most common form of thyroid carcinoma. It tends to present in the 3rd to 5th decade and shows a
modest female predominance. Despite its propensity for local lymphatic intrusion (which may cause multifocality of
tumor in the thyroid or cervical lymph node metastases), the tumor generally has an excellent prognosis with 90%
20-year survival.
Follicular carcinoma (choice A) is characterized by follicular cells and colloid on aspiration, and cannot be reliably
distinguished from thyroid adenoma.
Distinctive features of Hashimoto's disease (choice B) on aspiration are lymphocytes, plasma cells, and macrophages.
The most distinctive feature of medullary carcinoma (choice C) on aspiration is the presence of amyloid.
Thyroid adenoma (choice E) shows follicular cells and colloid on aspiration, and cannot be reliably distinguished from
follicular carcinoma.
--------------------------------------------------------------------------------20The correct answer is D. This is a straightforward
question relating to the definition of Nissl bodies. Rough endoplasmic reticulum present in neurons are called Nissl
bodies. They stain intensely with basic dyes and are found in the cell body and proximal dendrites, but not in the
axon hillock or axon.
--------------------------------------------------------------------------------21The correct answer is A. Sawtooth waves
appearing in bursts are associated with REM sleep.
Stage 1 (choice B) is associated with 4-7 Hz theta waves.
Stage 2 (choice C) is associated with 12-14 Hz sleep spindles and K-complexes.
Stage 3 (choice D) is associated with < 4 Hz, high-amplitude delta waves.
Stage 4 (choice E) is characterized by an EEG composed of about 50% delta waves.
Note that beta waves (15-18 Hz) occur during periods of more intense mental activity while awake. Alpha waves (8-
12 Hz) occur during awake, relaxed states. REM is the stage of sleep that most resembles the awake state on the
EEG.
--------------------------------------------------------------------------------22The correct answer is E. The disease is
poliomyelitis. Most infections with poliovirus cause only the influenza-like symptoms, but a small percentage progress
to paralytic poliomyelitis. The most common causes of death are aspiration and airway obstruction as a result of
bulbar paralysis and paralysis of respiratory muscles. Arrhythmias can also be life-threatening.
Acute renal failure (choice A) is usually not seen in poliomyelitis, although the bladder may become paralyzed.
Bowel paralysis (choice B) can be seen, but is not usually life-threatening.
Fulminant liver failure (choice C) is not a feature of poliomyelitis.
Gastrointestinal bleeding (choice D) can be seen in poliomyelitis, but is not usually life-threatening.
--------------------------------------------------------------------------------23The correct answer is D. The ratio of cells in
CELL BIOLOGY

bone marrow developing along myeloid lines to cells developing along erythroid lines is 3:1. An alternative way to
remember the normal marrow composition is that it typically contains about 60% granulocytes and their precursors;
20% erythroid precursors; 10% lymphocytes, monocytes, and their precursors; and 10% unidentified or
disintegrating cells. These numbers are worth remembering, because shifts away from normal values may be a
subtle clue to marrow abnormalities.
--------------------------------------------------------------------------------24The correct answer is E. An obese adult with
glucosuria, but not ketonuria, likely has type 2 diabetes mellitus. Type 2 diabetes is characterized by insulin
resistance resulting in hyperglycemia and increased serum osmolarity. The dehydration associated with osmotic
diuresis makes the hyperosmolarity worse. As the osmolarity increases above 330 mOsm/L, the osmotic loss of water
from neurons is sufficient to produce coma. In nonketotic, hyperosmolar coma, blood glucose values can range from
800 to 2,400 mg/dL and produce serum osmolarities of 330-440 mOsm/L.
Since type 2 diabetes is due to insulin resistance, plasma levels of insulin are usually normal to increased. Because b
cells secrete insulin and C-peptide in a 1:1 ratio, plasma concentration of C-peptide would also be normal to
increased (not decreased, choice A) in type 2 diabetes.
Even small amounts of insulin are sufficient to prevent ketosis. In type 2 diabetes there is enough insulin effect to
prevent significant lipolysis and subsequent formation of excess ketone bodies. Hence, acidosis (choice B) is not
typically associated with this disorder. Since b-hydroxybutyrate is a ketone body, its concentration in plasma is not
likely to be increased (choice D).
While there is a significant autoimmune component to type 1 diabetes, type 2 diabetes is not associated with
increased circulating antibodies (choice C) against b cell proteins such as glutamic acid decarboxylase.
--------------------------------------------------------------------------------25The correct answer is B. Neuromuscular
development is sufficient to allow fetal movement in the eighth week of life. Other features of week 8 include the
first appearance of a thin skin, a head as large as the rest of the body, forward-looking eyes, appearance of digits on
the hands and feet, appearance of testes and ovaries (but not distinguishable external genitalia), and a crown-rump
length of approximately 30 mm. By the end of the eighth week, nearly all adult structures have at least begun to
develop, and the fetus "looks like a baby".
--------------------------------------------------------------------------------26The correct answer is A. The Rb gene is an
example of a tumor suppressor gene. Tumor suppressor genes encode proteins that downregulate cell growth;
consequently, their deletion leads to the development of cells with a growth advantage over normal cells. Even if you
know nothing about the Rb protein, choice A is still the only logical answer because it is the only example of a
protein that, if absent, would favor cell growth. The Rb protein binds to transcription factors in the nucleus,
preventing cells from progressing from the S1 to M stages of the cell cycle. Children born with a 13q14 deletion have
only one chromosome encoding Rb; therefore only a single "hit " is required to completely knock out Rb production
and lead to the development of retinoblastoma. All of the incorrect choices are proteins encoded by oncogenes,
rather than tumor suppressor genes. Oncogenes favor tumorigenesis through overexpression, not deletion.
Growth factors (choice B) are oncoproteins that are produced by tumors and have a positive feedback effect.
Examples of growth factors are PDGF and fibroblast growth factor; the oncogenes encoding them are sis and hst-1,
respectively.
The prototypical growth factor-binding protein (choice C) is ras, which is mutated in a large variety of cancers. Ras
normally functions as an activator of protein kinases that regulate cell growth. Overactivity of the ras protein is highly
mitogenic.
Growth factor receptors (choice D) are either expressed as mutant forms or overexpressed in tumors, leading to
upregulation of growth. An example of a growth factor receptor oncogene is erb-B2, present in some breast cancers.
Transcription activators (choice E) are DNA-binding proteins that promote DNA transcription. Amplification of these
oncogenes causes cancer by promoting the transcription of growth-related genes.
--------------------------------------------------------------------------------27The correct answer is A. The obturator nerve
innervates the muscles of the medial compartment of the thigh. These include the adductor longus, adductor brevis,
adductor magnus, and gracilis muscles. The adductor magnus is also innervated by the tibial nerve.
The biceps femoris (choice B) is in the posterior compartment of the thigh. The long head of the biceps femoris is
innervated by the tibial portion of the sciatic nerve, and the short head of the biceps femoris is innervated by the
common peroneal portion of the sciatic nerve.
The rectus femoris (choice C) and vastus medialis (choice E) are two of the four heads of the quadriceps femoris
muscle. All four heads of the quadriceps femoris muscle are in the anterior compartment of the thigh, and are
innervated by the femoral nerve.
CELL BIOLOGY

The sartorius muscle (choice D) is in the anterior compartment of the thigh, and is innervated by the femoral nerve.
--------------------------------------------------------------------------------
28The correct answer is A. The patient has Burkitt's lymphoma. This type of lymphoma is a high-grade B-cell
lymphoma that occurs in endemic form in Africa (it is the most common neoplasm in children in an equatorial belt
that includes Africa and New Guinea) and sporadically in the United States and Europe. The sporadic form is often in
an abdominal site and occurs in young adults. The African form of Burkitt's lymphoma has been strongly associated
with antibodies directed against Epstein-Barr virus; the association is weaker in sporadic cases. A characteristic
translocation, t(8;14) (q24.l3;q32.33) has been described.
Hepatitis B (choice B) is associated with hepatocellular carcinoma. t(9;22) is the Philadelphia chromosome, which is
seen in some cases of CML and AML.
Herpesvirus (choice C) does not have a strong tumor association, although a link to cervical cancer has intermittently
been proposed. CD5 is a marker seen in small lymphocytic and mantle cell lymphomas.
HIV (choice D) is linked to Kaposi's sarcoma (and AIDS). Some patients also develop primary lymphomas (not usually
Burkitt's). CD4 is a marker for helper T cells and some T cell lymphomas.
Human papillomavirus (choice E) is linked with common warts, genital condylomata, and genital cancers. t(2;5) is
linked to anaplastic large cell lymphoma.
--------------------------------------------------------------------------------29The correct answer is E. The patient is
presenting with hypertension and signs and symptoms of benign prostatic hyperplasia (BPH). The essential diagnostic
characteristics of BPH include a decrease in the force and caliber of the urinary stream, nocturia, high post-void
residual volume, urinary retention, and azotemia. Terazosi

You might also like